0% found this document useful (0 votes)
1K views

Psat NMSQT Student Guide

Uploaded by

winniesaur
Copyright
© Attribution Non-Commercial (BY-NC)
Available Formats
Download as PDF, TXT or read online on Scribd
0% found this document useful (0 votes)
1K views

Psat NMSQT Student Guide

Uploaded by

winniesaur
Copyright
© Attribution Non-Commercial (BY-NC)
Available Formats
Download as PDF, TXT or read online on Scribd
You are on page 1/ 56

2010

Flip book over for


Test-taking Help

• Types of Test Questions


• Practice Test
(Separate Pull-out Booklet)

Official
Student Guide
to the

Preliminary SAT/National Merit Scholarship Qualifying Test

Scholarship Competitions
• National Merit®
Scholarship Program, page 2
• National Achievement®
Scholarship Program, page 20
conducted by
National Merit
Scholarship Corporation

www.nationalmerit.org
National Merit®
Scholarship Program
To enter the National Merit Scholarship Program and compete for recognition and 9,700
scholarships to be offered in 2012:

◾ Take the Preliminary SAT/National Merit Scholarship Qualifying Test


(PSAT/NMSQT ) in October 2010. ®
Program entrants must take the test in the specified year of the high school program
(see page 5). The 2010 PSAT/NMSQT is the qualifying test for entry to the 2012
program. Most entrants will complete high school and enroll in college in 2012.
◾ Meet other entry requirements.

Important information about the 2012 National Merit Scholarship Program:

Entry Requirements . . . . . . . . . . . . . . . . . . . . . . . . . . . . . . . 4
Program Recognition . . . . . . . . . . . . . . . . . . . . . . . . . . . . . . 5 Note: Information about
Types of Scholarships . . . . . . . . . . . . . . . . . . . . . . . . . . . . . . 7 participation in the
National Achievement®
Scholarship Sponsors Scholarship Program
Corporations and Business Organizations . . . . . . . . . . 8 begins on page 20.

Colleges and Universities. . . . . . . . . . . . . . . . . . . . . . . . 18

Want more information? Read on. ☛


Visit NMSC’s Web site: www.nationalmerit.org
Write: National Merit Scholarship Corporation (NMSC)
1560 Sherman Avenue, Suite 200
Evanston, IL 60201-4897
Telephone: (847) 866-5100

National Merit, National Achievement, Merit Scholarship, Achievement Scholarship, Merit Scholar, Achievement Scholar, and the corporate logo are federally registered
service marks of National Merit Scholarship Corporation. NMSC pages 1–23 © 2010 National Merit Scholarship Corporation. All rights reserved.

2 PSAT/NMSQT Official Student Guide: National Merit® Scholarship Program


Steps in the 2012 National Merit® Scholarship Competition

1,500,000 Entrants. In October 2010, U.S. high school students who take the Preliminary SAT/National
Merit Scholarship Qualifying Test (PSAT/NMSQT®) and meet other program requirements will enter the 2012
competition for National Merit Scholarship Program recognition and scholarships. Nearly all program participants
(entrants) will be juniors planning to enter college in 2012.

PARTICIPANTS WHO QUALIFY FOR NATIONAL MERIT PROGRAM RECOGNITION

50,000 High Scorers. NMSC uses PSAT/NMSQT Selection Index scores (the sum of critical reading,
mathematics, and writing skills scores) to determine participants who qualify for program recognition. In April
2011, high-scoring participants from every state will be offered the College Plans Reporting Service. These
students will be invited to name two colleges or universities to which they would like to be referred by NMSC.

34,000 Commended Students. In late 16,000 Semifinalists. In early September


September 2011, more than two-thirds of the 2011, nearly a third of the 50,000 high scorers will
50,000 high scorers will receive Letters of Com- be notified that they have qualified as Semifinalists.
mendation in recognition of their outstanding Semifinalists are the highest-scoring entrants in
academic promise, but they will not continue in the each state. NMSC will notify them through their
competition for National Merit Scholarships. How- schools and provide scholarship application mate-
ever, some of these students may be candidates for rials explaining requirements to advance in the
Special Scholarships provided by corporate and competition for National Merit Scholarships to be
business sponsors. offered in 2012.

15,000 Finalists. In February 2012, Semifinalists who meet


academic and other requirements will be notified that they have
advanced to Finalist standing. All National Merit Scholarship winners
(Merit Scholar® awardees) will be chosen from this group of Finalists,
based on their abilities, skills, and accomplishments.

SCHOLARSHIP WINNERS

1,300 Special Scholarship recipients. 8,400 Merit Scholarship® winners.


Beginning in March 2012, NMSC will notify win- Beginning in March 2012, NMSC will notify winners
ners of Special Scholarships provided by corpo- of the three types of National Merit Scholarships:
rate sponsors. Recipients, chosen from candidates • National Merit® $2500 Scholarships
sent scholarship application materials in December • Corporate-sponsored scholarships
2011, will be outstanding students (although • College-sponsored scholarships
not Finalists) who meet their sponsors’ eligibility In April, May, and July, NMSC will release names of
criteria. Sponsors will handle public announce- Merit Scholar designees to news media for public
ment of their Special Scholarship winners. announcement.

PSAT/NMSQT Official Student Guide: National Merit® Scholarship Program 3


The 2012 Competition 3. take the 2010 PSAT/NMSQT in the specified year of
the high school program and no later than the third year
Begins in 2010 in grades 9 through 12, regardless of grade classification or
educational pattern (explained in the next section).
®
The National Merit Scholarship Program is an annual
academic competition among high school students for A student’s responses to items “a” through “d” in
recognition and college scholarships. The program is Section 13 of the 2010 PSAT/NMSQT Answer Sheet (see
conducted by National Merit Scholarship Corporation below) are used to determine whether he or she meets
(NMSC), a not-for-profit organization that operates these requirements.
without government assistance.
The 2010 Preliminary SAT/National Merit Scholarship 13a. Are you enrolled as a high
school student?
®
Qualifying Test (PSAT/NMSQT ) is the qualifying test
for entry to the 2012 National Merit Program. The com-
Yes

petition will span about 18 months from entry in October No

2010 until the spring of 2012 when scholarships for college


13b. When will you complete or leave high
undergraduate study will be awarded. It is expected that school and enroll full time in college?
about 3.5 million students will take the PSAT/NMSQT 2011
in 2010, and over 1.5 million of them will meet require- 2012
2013
ments to enter this program. after 2013
Not planning to attend college

Entry Requirements 13c. How many years will you spend in


grades 9 through 12?
To enter the 2012 National Merit Program, a student must 4 years
meet all of the following requirements. He or she must: 3 years
2 years
1. be enrolled as a high school student, pro- 1 year
gressing normally toward graduation or completion of 5 or more years

high school by 2012, and planning to enter college no


later than the fall of 2012;
2. be a citizen of the United States; or be a U.S. lawful
permanent resident (or have applied for permanent resi-
dence, the application for which has not been denied) and
intend to become a U.S. citizen at the earliest opportunity
allowed by law (see below for documentation required
from scholarship candidates who have not yet become The NMSC Programs section of the PSAT/NMSQT Score
U.S. citizens); and Report shows the student’s responses to these questions
and whether entry requirements have been met.

Not yet a U.S. citizen?


If you qualify to continue in the 2012 competition (as a Semifinalist or Special Scholarship candidate) and you have not yet
become a U.S. citizen, you will be required to send the following documentation to NMSC with your Scholarship Application:
• A letter signed by you explaining your plans for becoming a citizen, including the date on which you expect to be eligible
and will apply for U.S. citizenship
AND
• If you are a U.S. lawful permanent resident, a copy of your Permanent Resident Card (Green Card); or a copy of
your passport including the identity/biographical page and the “I-551” stamp showing that you have been granted
lawful permanent resident status
OR
• If you have applied for U.S. lawful permanent residence, a copy of Form I-797 Receipt Notice from U.S. Citizenship
and Immigration Services (CIS) that verifies you filed the “Application to Register Permanent Residence or Adjust Status”
(Form I-485). Note: Form I-797 Receipt Notice must be received by NMSC on or before January 31, 2012.

4 PSAT/NMSQT Official Student Guide: National Merit® Scholarship Program


When to Take the Test student is participating in a postsecondary enrollment
options program and certifies the student’s status.
To participate in the National Merit Program, students
Note: If your high school standing and enrollment do not
must take the PSAT/NMSQT in the specified year of
fit one of the preceding descriptions (1, 2, or 3) because
their high school program. Because a student can
you plan to spend more than four years in grades 9
participate (and be considered for a scholarship) in only
through 12, or for any other reason, contact NMSC imme-
one specific competition year, the year in which the
diately about entry to the National Merit Program.
student takes the PSAT/NMSQT to enter the competition
is very important.
1. Students who plan to spend the usual four Unable to Take the PSAT/NMSQT?
years in high school (grades 9 through 12) before If you do not take the 2010 PSAT/NMSQT because
entering college full time must take the qualifying test of illness, an emergency, or other extenuating cir-
in their third year of high school (grade 11, junior year). cumstance, you may still be able to enter NMSC’s
2012 competitions. To request information about
another route of entry after the October 2010 PSAT/
Sophomores who take the 2010 PSAT/NMSQT
NMSQT administration, WRITE to NMSC as soon
but plan to spend four years in grades 9 through
as possible but no later than March 1, 2011. Do not
12 will not meet entry requirements for the 2012
National Merit Program. They must take the PSAT/ delay; the earlier you write, the more options you
NMSQT again in 2011 (when they are juniors) to will have for scheduling test dates.
enter the competition that will end when scholar- Mail your request to:
ships are awarded in 2013, the year they will com- National Merit Scholarship Corporation
plete high school and enter college. Attn: Educational Services
1560 Sherman Avenue, Suite 200
Evanston, IL 60201-4897
2. Students who plan to leave high school early to Your letter must be postmarked on or before March 1,
enroll in college full time, after spending three years or
2011 for your request to be considered.
less in grades 9 through 12, usually can participate in the
National Merit Program if they take the PSAT/NMSQT
before they enroll in college. To enter the 2012 competi- Program Recognition
tion, these students must be in either the next-to-last or NMSC uses PSAT/NMSQT Selection Index scores (the
the last year of high school when they take the 2010 sum of the critical reading, mathematics, and writing
PSAT/NMSQT: skills scores) as an initial screen of more than 1.5 million
a) if they are in the next-to-last year of high school program entrants. The 2010 test scores of all students
when they take the 2010 PSAT/NMSQT, they will who meet entry requirements for the 2012 program will
be finishing their last high school year when be considered. Entrants whose scores qualify them for
awards are offered in 2012; or recognition will be contacted by NMSC through their
high schools.
b) if they are in their last year of high school when
they take the 2010 PSAT/NMSQT, they will be College Plans Reporting Service. Approximately 50,000
completing their first year of college when schol- program participants with the highest Selection Index
arships are awarded in 2012. scores will be offered the College Plans Reporting Service.
In April 2011, NMSC will contact these high scorers
3. Students who plan to participate in a post- through their high schools and give them an opportunity
secondary enrollment options program (through which to be referred to two U.S. colleges or universities of their
they enroll simultaneously in both high school and col- choice. Then in late July, NMSC will send information to
lege) must take the qualifying test in their third year of the two institutions named by each student who responds.
high school (grade 11, junior year). To enter the compe- The purpose of this free and optional referral service is to
tition that ends when scholarships are offered in 2012, encourage college officials to contact these scholastically
these students must be in their third year of high school talented students about opportunities for admission and
when they take the 2010 PSAT/NMSQT, the same as all financial aid.
other students who plan to spend four years in grades
9 through 12. The high school determines whether a

PSAT/NMSQT Official Student Guide: National Merit® Scholarship Program 5


Commended Students. In September 2011, more than and U.S. boarding schools that enroll a sizable propor-
two-thirds (about 34,000) of the high scorers will be tion of their students from outside the state in which the
designated Commended Students. They will be named school is located. A participant can be considered for
on the basis of a nationally applied Selection Index Semifinalist standing in only one state or selection unit,
qualifying score that may vary from year to year. This based on the high school in which the student is regularly
qualifying score is generally within the 96th percentile of enrolled when taking the PSAT/NMSQT.
college-bound juniors who take the PSAT/NMSQT. Finalists. A Semifinalist must fulfill several additional
In recognition of their outstanding ability and poten- requirements and advance to the Finalist level of the
tial for academic success in collegge, these students will be competition before being considered for a National Merit
honored with Letters of Commendation sent to them Scholarship. Approximately 90 percent (about 15,000) of
through their high schools. Although Commended the Semifinalists are expected to become Finalists and
Students will not continue in the competition for National receive a Certificate of Merit attesting to their distin-
Merit Scholarships, some may be candidates for Special guished performance in the competition.
Scholarships offered by corporate sponsors (see page 8).
NMSC will contact those candidates in December 2011. To qualify as a Finalist, a Semifinalist must:
Semifinalists. Some 16,000 of the high scorers, repre- 1. continue to meet all program entry requirements
senting less than 1 percent of the nation’s high school gradu- (explained on page 4);
ating seniors, will qualify as Semifinalists. Only 2. be enrolled in the last year of high school and
Semifinalists will have an opportunity to advance in the planning to enroll full time in college the following
®
competition for Merit Scholarship awards. NMSC will
notify Semifinalists of their standing and send scholar-
fall, or be enrolled in the first year of college if grades 9
through 12 were completed in three years or less;
ship application materials to them through their high 3. complete the National Merit Scholarship
schools in September 2011. Their names will be sent to Application with all information requested, which
U.S. colleges and released to news media for public includes writing an essay;
announcement in mid-September. 4. have a record of very high academic performance
NMSC designates Semifinalists on a state represen- in all of grades 9 through 12 and in any college course
tational basis to ensure that academically able young work taken (the high school must provide a complete
people from all parts of the United States are included record of courses taken and grades earned by the student,
in this talent pool. Using the latest data available, an as well as information about the school’s curriculum and
allocation of Semifinalists is determined for each state, grading system);
based on the state’s percentage of the national total of
5. be fully endorsed for Finalist standing and rec-
high school graduating seniors. For example, the number
ommended for a National Merit Scholarship by the high
of Semifinalists in a state that enrolls approximately two school principal;
percent of the nation’s graduating seniors would be about
320 (2 percent of the 16,000 Semifinalists). ®
6. take the SAT and earn scores that confirm the
PSAT/NMSQT performance that resulted in Semifinalist
NMSC then arranges the Selection Index scores of all
standing; and
National Merit Program participants within a state in
descending order. The score at which a state’s allocation 7. provide any other documentation and information
is most closely filled becomes the Semifinalist qualify- that NMSC requests.
ing score. Entrants with a Selection Index at or above
the qualifying score are named Semifinalists. As a result Only Finalists will be considered for the 8,400 National
of this process, Semifinalist qualifying scores vary from Merit Scholarships. Before receiving an award, a Finalist
state to state and from year to year, but the scores of all must (a) notify NMSC of plans to enroll in a college or
Semifinalists are extremely high. university in the United States that holds accredited status
In addition to Semifinalists designated in each of the with a regional accrediting commission on higher educa-
50 states and without affecting the allocation to any tion, and (b) plan to enroll full time in an undergraduate
course of study leading to a traditional baccalaureate
state, Semifinalists are named in several other selection
degree. NMSC scholarship stipends are not payable for
units that NMSC establishes for the competition. These
attendance at service academies, virtual universities, and
units are for students attending schools in the District of
certain institutions that are limited in their purposes or
Columbia, schools in U.S. commonwealths and territo-
training.
ries, schools in other countries that enroll U.S. citizens,

6 PSAT/NMSQT Official Student Guide: National Merit® Scholarship Program


Choosing Scholarship Types of Scholarships
Winners Some 8,400 National Merit Scholarships of three types
Approximately half of the Finalists will be Merit and approximately 1,300 Special Scholarships will be
awarded in 2012; these 9,700 awards will have a com-
®
Scholarship winners (Merit Scholar awardees). Winners
are chosen on the basis of their abilities, skills, and bined value of more than $48 million. Different types of
accomplishments—without regard to gender, race, ethnic scholarships will be offered, but no student can receive
origin, or religious preference. Scholarship recipients are more than one monetary award from NMSC.
the candidates judged to have the greatest potential for
success in rigorous college studies and beyond.
®
National Merit $2500 Scholarships. These awards are
unique because every Finalist is considered for one and
The selection process involves evaluating substantial winners are named in every state and other selection
amounts of information about Finalists obtained from units. The number awarded in each state is determined by
both students and their high schools. Included are the the same representational procedure used to designate
Finalist’s academic record (course load and difficulty Semifinalists. Finalists compete with all other Finalists in
level, depth and breadth of subjects studied, and grades their state or selection unit for one of the 2,500 National
earned); standardized test scores; the student’s essay; Merit $2500 Scholarships. Winners are selected by a com-
demonstrated leadership and contributions to school and mittee of college admission officers and high school
community activities; and the school official’s written counselors.
recommendation and characterization of the Finalist. National Merit $2500 Scholarships provide a single
The same process is used to select Special Scholarship payment of $2,500. NMSC’s own funds support the
winners for a corporate sponsor’s awards. majority of these scholarships, but corporate sponsors
help underwrite these awards with grants they provide to
NMSC in lieu of paying administrative fees.

A Total of 9,700 Scholarships Offered


National Merit $2500 Corporate-sponsored Corporate-sponsored College-sponsored
Scholarships Merit Scholarships Special Scholarships Merit Scholarships
Who is All Finalists compete Finalists who meet High performing Finalists who plan
considered? with all other Finalists criteria specified by program partici- to attend a spon-
in their state or other a corporate sponsor, pants (although not sor college and have
selection unit. usually: Finalists) who meet informed NMSC that
• children of a sponsor’s criteria; the sponsor college is
employees; most are for: their first choice
• residents of specific • children of
communities; or employees;
• Finalists with cer- • residents of specific
tain college major communities; or
or career plans • participants with
certain college
major or career
plans
Who selects A committee of college NMSC’s professional NMSC’s professional Officials of each
winners? admission officers and staff staff sponsor college
high school counselors
What is the Awards provide a Varies by sponsor— Varies by sponsor— Awards are renewable
monetary value? one-time payment of awards can be one- awards can be one- for 4 years of study at
$2,500. time or renewable (See time or renewable (See the sponsor institu-
chart on page 8.) chart on page 8.) tion. Stipends range
from $500 to $2,000
per year.
When does NMSC Late March Early March Early March Late April
begin sending
scholarship offers?

PSAT/NMSQT Official Student Guide: National Merit® Scholarship Program 7


Corporate-sponsored scholarships. In 2010, about 250 it is canceled if a winner does not attend the college
corporations, company foundations, and businesses spon- financing the award.
sored scholarships through the National Merit Program Officials of each sponsor institution choose award
as part of their educational philanthropy. These spon- recipients from among Finalists who have been accepted
sors committed approximately $19 million to support for admission and have informed NMSC that the sponsor
more than 1,000 corporate-sponsored Merit Scholarship college or university is their first choice. College officials
awards and 1,300 Special Scholarships for students with also determine each winner’s stipend within a range of
qualifications that particularly interest them. $500 to $2,000 per year. The college may meet some of a
The number of National Merit Scholarships a com- winner’s financial need with an aid package that includes
pany or business offers annually may range from one to loans, employment, and grants; however, the Merit
more than 100. Finalists who meet a sponsor’s criteria are Scholarship stipend must represent at least half the win-
identified from information Semifinalists supply on their ner’s need, up to the annual stipend maximum of $2,000,
scholarship applications. Winners are selected from this unless the student’s total need (as calculated by the col-
pool based on their abilities, skills, and accomplishments. lege) is met with gift aid.
The scholarship name usually identifies the grantor—for
example, National Merit XYZ Company Scholarship.
Over two-thirds of the program’s corporate sponsors
Scholarship Sponsors
also provide Special Scholarships. Organizations that Approximately 450 independent organizations and insti-
sponsor Special Scholarships make Entry Forms available tutions sponsor over two-thirds of the National Merit
to children of employees or members, or to students with Scholarships offered each year. All Special Scholarships
other qualifications that interest them. Entry Forms are are sponsored by business organizations that also support
completed by students (and their parents, if applicable). If Merit Scholarship awards.
the number of Finalists eligible for a sponsor’s awards is Corporate scholarship sponsors. Following is a list of
smaller than the number of awards the corporate organi- corporate organizations that currently sponsor scholar-
zation wishes to offer, NMSC establishes a pool of high ships in the National Merit Program with any eligibility
scoring candidates who filed Entry Forms. NMSC then criteria that apply to candidates for their awards. An
sends Special Scholarship application materials to these asterisk (*) indicates the sponsor offers Special Scholar-
candidates through their high schools and invites them to ships in addition to National Merit Scholarships and
compete for the sponsor’s Special Scholarships. requires that Entry Forms be filed. The number of awards
Each corporate sponsor specifies the monetary limits shown is an estimate of the yearly total, and neither the
of scholarships it finances and decides whether the awards sponsor nor NMSC is obligated to offer these scholarships
provide one-time payments or stipends that can be in 2011, 2012, or beyond.
renewed for up to four years of college undergraduate ■
study. Although financial need is not considered in the
selection of winners, some corporate-sponsored renew-
* Accenture LLP–6 awards for children of employees
able scholarships provide variable stipends that are indi-
of Accenture LLP.
vidually determined, taking into account college costs
and family financial circumstances. * The Acushnet Company, Inc.–3 awards for children
of employees of the company and its subsidiaries.
Type of Award Type of Stipend Stipend Ranges
Renewable Variable $500–$10,000 per year * Administaff Services, L.P.–4 awards—2 awards for
Renewable Fixed $1,000–$5,000 per year children of corporate employees; 2 awards for children
One-time Single-payment $2,500–$5,000 of worksite employees.

* ADP Foundation–37 awards for children of employees


College-sponsored Merit Scholarships. In the 2012
of Automatic Data Processing, Inc.
competition, it is expected that over 4,900 National Merit
Scholarships will be offered to Finalists who plan to Affymetrix, Inc.–2 awards for Finalists who are children
attend a sponsor college or university. (See the list of some of employees of the corporation and its subsidiaries.
200 colleges that currently are Merit Scholarship sponsors
beginning on page 18.) A college-sponsored scholarship is
renewable for up to four years of undergraduate study at * This corporate sponsor offers Special Scholarships
the sponsor institution. It cannot be transferred; therefore, ®
in addition to Merit Scholarship awards, and Entry
Forms that the sponsor makes available must be filed.

8 PSAT/NMSQT Official Student Guide: National Merit® Scholarship Program


Air Products and Chemicals, Inc.–Every Finalist who * Arch Chemicals, Inc.–3 awards for children of
is the child of an employee of the corporation or a sub- employees of the corporation and its subsidiaries.
sidiary will be offered an award.
* Archer Daniels Midland Company–10 awards for
* Akzo Nobel Inc.–6 awards for children of employees of children of employees of ADM and its subsidiaries.
the company and its subsidiaries.
* Arkema Inc. Foundation–4 awards for children of
* Albany International–2 awards for children of employees of Arkema Inc. and its subsidiaries.
employees of the corporation and its subsidiaries.
* Armstrong Foundation–Up to 25 awards for children
The Alcon Foundation, Inc.–Every Finalist who is the of employees of Armstrong World Industries, Inc. and
child of an employee of Alcon Laboratories, Inc. or an its subsidiaries.
affiliate will be offered an award.
ASC Partners, LLC–2 awards for Finalists in the
* The Allergan Foundation–6 awards for children of National Merit Scholarship Program.
employees of Allergan, Inc. and its eligible subsidiaries.
ASM Materials Education Foundation–1 award for a
* Ameren Corporation Charitable Trust–4 awards for Finalist who plans a career in the field of materials
children of employees of Ameren Corporation. engineering.

* American City Business Journals, Inc.–2 awards for * Astellas US LLC–2 awards for children of employees of
children of employees of the corporation. the corporation.

American Electric Power Company, Inc.–5 awards for * AstraZeneca Pharmaceuticals LP–10 awards for chil-
Finalists who are children of employees of the corpora- dren of employees of the corporation.
tion and its subsidiaries.

* American Financial Group–2 awards for children of
employees of the corporation and its subsidiaries. * BASF Corporation–12 awards for children of
employees of the corporation and its subsidiaries.
* AmerisourceBergen Corporation–8 awards for
children of employees of the corporation and its Battelle–Every Finalist who is the child of an employee
subsidiaries. of the company will be offered an award.
* The AMETEK Foundation–7 awards for children of Bayer USA Foundation–5 awards for Finalists who are
employees of AMETEK, Inc. and its subsidiaries. children of employees of Bayer Corporation and its
subsidiaries.
* Anadarko Petroleum Corporation–4 awards for
children of employees of the corporation and its Mary E. Beyerle Trust–7 awards for Finalists from
subsidiaries. Maryland, New Jersey, and Pennsylvania.
Anheuser-Busch Companies, Inc.–1 award for a * The Black & Decker Corporation–10 awards for
Finalist who is a child of an employee of the corpora- children of employees of the corporation and its
tion or one of its subsidiaries. subsidiaries.
* Aon Foundation–8 awards for children of employees Black & Veatch Corporation–2 awards for Finalists
of Aon Corporation and its subsidiaries. who are children of employees of the corporation and
Apache Corporation–2 awards for Finalists who its subsidiaries.
are children of employees of the corporation and its * BMC Software, Inc.–6 awards for children of
subsidiaries. employees of the corporation and its subsidiaries.

®
* This corporate sponsor offers Special Scholarships in addition to Merit Scholarship awards, and Entry Forms that the sponsor
makes available must be filed.

PSAT/NMSQT Official Student Guide: National Merit® Scholarship Program 9


Corporate scholarship sponsors * Cargill, Incorporated–10 awards for children of
(continued) employees of the corporation and its subsidiaries and
BNSF Foundation–Up to 10 awards for Finalists who affiliated companies.
are children of employees of BNSF Corporation and its
* Carlisle Companies Incorporated–2 awards for chil-
subsidiaries. dren of employees of the corporation and its subsidiaries.
The Boeing Company–Every Finalist who is the
* Centocor Ortho Biotech Inc.–3 awards for children of
child of an employee of the company or a subsidiary employees of U.S. Centocor, Ortho Biotech Inc. and JJPS.
will be offered an award.
Chevron Corporation–10 awards—8 awards for Final-
* BorgWarner Inc.–6 awards for children of employees ists who are children of employees of the corporation
of the corporation and its divisions and subsidiaries. and its affiliated companies; 2 awards for other Finalists
* BP Foundation, Inc.–Up to 50 awards for children planning to major in geological and earth sciences/
of employees and annuitants of BP, plc and its wholly geosciences, geological/geophysical, or petroleum
owned subsidiaries. engineering.

* Branch Banking & Trust Company–10 awards for * CNA Surety–2 awards for children of employees of the
children of employees of BB&T. company, and its subsidiaries and affiliates.

* Bridgestone Americas Trust Fund–Up to 50 awards * Colgate-Palmolive Company–10 awards for children
for children of employees of Bridgestone Americas, Inc. of employees of the company and its subsidiaries.
and its participating subsidiaries.
* Collective Brands, Inc.–5 awards for children of
* Bristol-Myers Squibb Foundation, Inc.–50 awards employees of the corporation and its subsidiaries.
for children of employees of the Bristol-Myers Squibb Computer Sciences Corporation–5 awards for
Company’s U.S. divisions and subsidiaries. Finalists who are children of employees of the corpora-
* Broadridge Financial Solutions, Inc.– 4 awards for tion and its subsidiaries.
children of employees of the corporation and its quali- ConAgra Foods Foundation–Up to 10 awards for
fying subsidiaries. Finalists who are children of employees of ConAgra
* Bunge North America, Inc.–4 awards for children of Foods, Inc. and its subsidiaries.
employees of the corporation and its subsidiaries; the
* CONSOL Energy Inc.–16 awards for children of
awards are distributed among employment groups. employees of the corporation and its designated
* Bunzl USA, Inc.–5 awards for children of employees of subsidiaries.
the corporation and its subsidiaries.
* Continental Grain Foundation–2 awards for children
of employees of Continental Grain Company.

* Corning Incorporated–5 awards for children of
employees of the corporation and its subsidiaries.
* CACI International Inc–5 awards—3 awards for chil-
dren of employees of the company and its subsidiaries; * COUNTRY Financial–5 awards for children of
2 awards for Finalists who are planning to major in COUNTRY employees.
certain technological and mathematical fields.
* Covidien–20 awards for children of employees of
Cardinal Health Foundation–Every Finalist who is Covidien and its subsidiaries.
the child of an employee of Cardinal Health, Inc. or a
participating affiliate will be offered an award. * Cytec Industries Inc.–4 awards for children of
employees of the corporation and its subsidiaries.

®
* This corporate sponsor offers Special Scholarships in addition to Merit Scholarship awards, and Entry Forms that the sponsor
makes available must be filed.

10 PSAT/NMSQT Official Student Guide: National Merit® Scholarship Program


■ * FedEx Freight Corporation–20 awards for children of
employees of the corporation.
* Dole Food Company, Inc.–2 awards for children of * FedEx Ground Package System, Inc.–6 awards for
employees of the corporation and its subsidiaries. children of employees of the corporation.
* R. R. Donnelley Foundation–16 awards for children * Ferro Foundation–2 awards for children of employees
of employees of the company and its subsidiaries. of Ferro Corporation.
The Dow Chemical Company Foundation–15 awards * Fifth Third Foundation–17 awards for children of
for Finalists who are children of employees of the com- employees of Fifth Third Bank.
pany and its subsidiaries.
* FIL Holdings Corporation Inc.–2 awards for children
* Dow Jones Foundation–Up to 25 awards for children of employees of the corporation and its subsidiaries.
of employees of Dow Jones & Company, Inc. and its
wholly owned subsidiaries. * FMC Corporation–3 awards for children of employees
of the corporation and its subsidiaries.
* Dresser, Inc.–2 awards for children of employees of the
corporation and its business units. * FMC Technologies, Inc.–2 awards for children of
employees of the corporation and its affiliated businesses.

Formosa Plastics Corporation, U.S.A.–Every Finalist
who is the child of an employee of the corporation or
E*TRADE Financial–2 awards for Finalists who are an affiliated organization will be offered an award.
children of employees of the corporation.
* FPL Group Foundation–10 awards for children of
Eastman Chemical Company Foundation, Inc.–8 employees of FPL Group and its subsidiaries.
awards for Finalists who are children of employees of
Eastman Chemical Company and its subsidiaries. ■
* Eaton Charitable Fund–15 awards for children of
employees of Eaton Corporation. * GAF Materials Corporation–2 awards for children of
employees of the corporation and its subsidiaries; the
* El Paso Corporation–20 awards for children of awards are distributed among divisions.
employees of the corporation and its subsidiaries.
* Gannett Foundation, Inc.–12 awards for children of
* Electrolux North America–2 awards for children of employees of Gannett Co. Inc.
employees of the corporation and its subsidiaries.
GATX Corporation–2 awards for Finalists who are child-
* Estee Lauder Companies, Inc.–3 awards for children ren of employees of the corporation and its subsidiaries.
of employees of the corporation and its U.S. affiliates
and subsidiaries. GEICO Philanthropic Foundation–3 awards for
Finalists who are children of associates of GEICO
* ETHICON, INC.–2 awards for children of employees of Corporation and its subsidiaries.
the corporation.
* GenCorp Foundation, Incorporated–5 awards for
■ children of employees of GenCorp and its subsidiaries.

General Dynamics–Every Finalist who is the child of


* FedEx Custom Critical, Inc.–4 awards—2 awards an employee of the corporation or a subsidiary will be
for children of employees; 2 awards for children and offered an award.
grandchildren of contractors.

®
* This corporate sponsor offers Special Scholarships in addition to Merit Scholarship awards, and Entry Forms that the sponsor
makes available must be filed.

PSAT/NMSQT Official Student Guide: National Merit® Scholarship Program 11


Corporate scholarship sponsors ■
(continued)

* General Dynamics, Ordnance and Tactical Systems– Harris Corporation–2 awards for Finalists who are
2 awards for children of employees of the corporation children of employees of the corporation and its
and its subsidiaries. divisions.
General Mills Foundation–Up to 10 awards for * Harsco Corporation Fund–Up to 12 awards for child-
Finalists who are children of employees of General of employees of the corporation and its subsidiaries.
Mills, Inc. and its subsidiaries.
* H. J. Heinz Company Foundation–6 awards for chil-
* Georgia-Pacific Foundation, Inc.–50 awards for chil- dren of employees of the company and its affiliates.
dren of employees of Georgia-Pacific LLC and its
subsidiaries within the United States; the awards are * Henkel of America, Inc.–6 awards for children of
distributed among divisions. employees of the corporation and its subsidiaries.

* Gerdau Ameristeel Corporation–10 awards for chil- * Hoffmann-La Roche Inc.–8 awards for children
of employees of the corporation and its designated
dren of employees of the corporation.
subsidiaries.
* GKN Foundation–5 awards for children of employees
Honeywell International, Inc.–25 awards for Finalists
of GKN America Corp. and its subsidiaries.
who are children of employees of the corporation and
* Gleason Foundation–Up to 8 awards for children its subsidiaries.
of employees of Gleason Corporation and its U.S.
subsidiaries and divisions. * Hormel Foods Charitable Trust–18 awards for chil-
dren of employees of Hormel Foods Corporation and
* Global Pharmaceutical Supply Group–4 awards for its subsidiaries.
children of employees of the organization.
* Houghton Mifflin Harcourt Publishing Company–2
* Global Tungsten & Powders Corp.–2 awards for awards for children of employees of the company and
children of employees of the corporation. its subsidiaries.

Goodrich Foundation, Inc.–Every Finalist who is the * The Harvey Hubbell Foundation–4 awards for
child of an employee of Goodrich Corporation or a children of employees of Hubbell Incorporated and
subsidiary will be offered an award. its subsidiaries.

* The Goodyear Tire & Rubber Company–8 awards for ■


children of employees of the company.

* W.W. Grainger, Inc.–10 awards for children of employ- * The IKON Office Solutions Foundation–10 awards for
ees of the corporation and its U.S. subsidiaries. children of employees of IKON Office Solutions, Inc.
and its subsidiaries.
* Greyhound Lines, Inc.–Up to 6 awards for children of
employees of the corporation and its subsidiaries. * Illinois Tool Works Foundation–30 awards for children
of employees of Illinois Tool Works Inc. and its subsidiaries.
* (Gulfstream Aerospace Corporation) Rolls-Royce
North America Inc.–2 awards for children of employees * Ingersoll-Rand Charitable Foundation–17 awards for
of Gulfstream Aerospace Corporation. children of employees of Ingersoll Rand Company and
its subsidiaries.

®
* This corporate sponsor offers Special Scholarships in addition to Merit Scholarship awards, and Entry Forms that the sponsor
makes available must be filed.

12 PSAT/NMSQT Official Student Guide: National Merit® Scholarship Program


* Intermec Foundation–10 awards for children of ■
employees of Intermec Technologies Corporation.

* International Specialty Products Inc.–2 awards for * LANXESS Corporation–2 awards for children of
children of employees of the corporation and its sub- employees of the corporation and its subsidiaries.
sidiaries; the awards are distributed among divisions.
* Lennox International Inc.–20 awards for children of
* International Union of Bricklayers and Allied employees of the corporation.
Craftworkers–3 awards for children of members of the
union in good standing. * Liberty Mutual Scholarship Foundation–16 awards
for children of employees of Liberty Mutual Insurance
* Invensys Controls–2 awards for children of employees Company and its subsidiaries.
of Invensys Controls and its subsidiaries.
* Lockheed Martin Corporation Foundation–100
awards for children of employees of Lockheed Martin
■ and its subsidiaries.

Jacobs Engineering Foundation–Every Finalist who * Loews Foundation–4 awards for children of employees
is the dependent of an employee of Jacobs Engineering of Loews Corporation and its divisions and wholly
Group, Inc. or a subsidiary will be offered an award. owned subsidiaries.

* John Bean Technologies Corporation–2 awards for * Lord & Taylor Foundation–2 awards for student asso-
children of employees of JBT Corporation and its ciates and children of associates of Lord & Taylor LLC.
subsidiaries.
* Lorillard Tobacco Company–8 awards for children of
* Johnson & Johnson Consumer Companies, Inc.–6 employees of the company and its subsidiaries.
awards for children of employees of the corporation
The Lubrizol Foundation–Every Finalist who is the
and its participating companies.
child of an employee of The Lubrizol Corporation or a
* Johnson & Johnson Pharmaceutical Research & U.S. subsidiary will be offered an award.
Development, L.L.C.–3 awards for children of employ-
ees of the corporation. * Luxottica Retail–8 awards for children of employees of
the corporation.
* Johnson & Johnson World Headquarters–2 awards
for children of employees of Johnson & Johnson World ■
Headquarters.
* Macy’s Inc.–54 awards for student employees and
■ children of employees of Macy’s Inc. and its direct and
indirect subsidiaries; the awards are distributed among
* Kaman Corporation–3 awards for children of divisions.
employees of the corporation and its subsidiaries.
* Marsh & McLennan Companies, Inc.–20 awards
* The Kennametal Foundation–5 awards for children of for children of employees of the corporation and its
employees of Kennametal Inc. and its subsidiaries. subsidiaries.

Knovel–2 awards for Finalists planning to major in McDonald’s Corporation–5 awards for Finalists who
mechanical, design, or civil engineering. are children of employees of the corporation and its
subsidiaries.

®
* This corporate sponsor offers Special Scholarships in addition to Merit Scholarship awards, and Entry Forms that the sponsor
makes available must be filed.

PSAT/NMSQT Official Student Guide: National Merit® Scholarship Program 13


Corporate scholarship sponsors Navistar Foundation–1 award for a Finalist who is a
(continued) child of an employee of the Navistar International
Corporation.
* The McGraw-Hill Companies, Inc.–20 awards for
children of employees of the corporation and its
* New Jersey Manufacturers Insurance Group–At least
subsidiaries. 2 awards for children of employees of New Jersey
Manufacturers Insurance Group.
* McKesson Foundation Inc.–20 awards for children
of employees of McKesson and its divisions and New York Life Foundation–Every Finalist who is the
subsidiaries. child of an employee or agent of New York Life or cer-
Mead Witter Foundation, Inc.–10 awards for Finalists tain subsidiaries will be offered an award.
from specified geographic areas of Wisconsin.
* NewMarket Corporation–2 awards for children of
Medline Industries, Inc.–2 awards for Finalists who employees of the corporation and its subsidiaries.
are children of employees of the corporation.
* NiSource Charitable Foundation–15 awards for chil-
Glenn and Ruth Mengle Foundation–Up to 4 awards dren of employees of NiSource and its subsidiaries.
for Finalists from three Pennsylvania counties—
* Norfolk Southern Foundation–10 awards for children
Clearfield, Elk, and Jefferson. of employees of Norfolk Southern Corporation and its
MetLife Foundation–Every Finalist who is the child of affiliated companies.
an employee of MetLife will be offered an award.
* Northrop Grumman Corporation–60 awards for chil-
The MITRE Corporation–2 awards for Finalists who dren of employees of the corporation and its subsidiaries.
are children of employees of the corporation.
* Novartis Corporation–20 awards for children of
employees of the corporation and its divisions and
* The Modine Manufacturing Company Foundation,
Inc.–2 awards for children of employees of Modine subsidiaries.
Manufacturing Company and its divisions and
* Novo Nordisk Inc.–2 awards for children of employees
subsidiaries. of the corporation.
* The Moody’s Foundation–2 awards for children of * NSTAR Foundation–2 awards for children of employees
employees of Moody’s Corporation and its subsidiaries. of NSTAR and its subsidiaries.
Motorola Foundation–Every Finalist who is the child
of an employee of Motorola Inc. or a subsidiary will be ■
offered an award.
Occidental Petroleum Corporation–Every Finalist
■ who is the child of an employee of the corporation or a
division or subsidiary will be offered an award.
National Distillers Distributors Foundation–7 awards O’Donnell Foundation–2 awards for Finalists from
for Finalists in the National Merit Scholarship Program. specified high schools located in Texas.
* (National Gypsum Company) C. D. Spangler * Old National Bank Foundation–3 awards for chil-
Foundation, Inc.–2 awards for children of employees dren of associates of Old National Bancorp and its entities.
of National Gypsum Company and its subsidiaries.
Olin Corporation Charitable Trust–Every Finalist
* Nationwide Foundation–10 awards—6 awards for chil- who is the child of an employee of Olin Corporation or
dren of employees of Nationwide Insurance Companies a subsidiary will be offered an award.
and certain affiliates and associates; 4 awards for chil-
dren of career agents.

®
* This corporate sponsor offers Special Scholarships in addition to Merit Scholarship awards, and Entry Forms that the sponsor
makes available must be filed.

14 PSAT/NMSQT Official Student Guide: National Merit® Scholarship Program


* OMNOVA Solutions Foundation, Inc.–3 awards for PPL–Every Finalist who is the child of an employee of
children of employees of OMNOVA Solutions, Inc. and the corporation will be offered an award.
its subsidiaries.
Public Service Enterprise Group–Every Finalist who
Omron Foundation, Inc.–5 awards for Finalists with is the child of an employee of Public Service Enterprise
preference (1st) for those who are children of employ- Group or its subsidiaries will be offered an award.
ees of Omron; (2nd) for those from Illinois or Michigan
intending to major in electrical engineering, science, or * Putnam LLC–2 awards for children of employees of
mathematics. Putnam Investments and its subsidiaries.

* Ortho-McNeil-Janssen Pharmaceuticals, Inc.–3 awards * Quanex Foundation–3 awards for children of employ-
for children of employees of the corporation and its ees of Quanex Building Products and its divisions and
subsidiaries. subsidiaries.

Owens Corning Foundation–Every Finalist who is the ■


child of an employee of Owens Corning will be offered
an award.
* Research Triangle Institute–2 awards for children of
employees of the institute.

* Retail Brand Alliance, Inc.–2 awards for children of
Parker Hannifin Foundation–Every Finalist who is the employees of the corporation and its subsidiaries.
child of an employee of Parker Hannifin Corporation
* Rexam Inc. Foundation–2 awards for children of
or a subsidiary will be offered an award. employees of Rexam Inc. and its subsidiaries.
* Frank E. Payne and Seba B. Payne Foundation–3 * Reynolds American Foundation–20 awards for chil-
awards for children or grandchildren of employees of dren of employees of Reynolds American Inc. and its
John Crane Inc. and its domestic subsidiaries. eligible subsidiaries.
* The Penn Mutual Life Insurance Company–2 awards * Rheem Manufacturing Company–5 awards for children
for children of company employees, agents, and office of employees of the company and its subsidiaries.
employees.
* Robbins & Myers Foundation–1 award for a child of
PepsiCo Foundation, Inc.–Every Finalist who is the an employee of Robbins & Myers, Inc. or one of its
child of an employee of PepsiCo, Inc. or a division or subsidiaries.
subsidiary will be offered an award.
Rockwell Automation Charitable Corporation–5
* Pfizer Inc–50 awards for children of employees of the awards for Finalists who are children of employees of
corporation and its subsidiaries. Rockwell Automation and its subsidiaries.
* Pilkington North America, Inc.–2 awards for children Rockwell Collins Charitable Corporation–8 awards
of employees of the corporation and its subsidiaries. for Finalists who are children of employees of Rockwell
Collins and its subsidiaries.
* PPG Industries Foundation–68 awards—40 awards
for children of employees of PPG Industries, Inc. and Rolls-Royce North America Inc.–3 awards for Finalists
its subsidiaries; 28 awards for residents of communities who are children of employees of Rolls-Royce plc and
where PPG has operations. eligible subsidiaries.
* PPG Industries, Inc.–2 awards for children of employ- Ryerson Foundation–Every Finalist who is the child
ees of Platinum Distributors of PPG Industries, Inc. of an employee of Ryerson Inc. or a subsidiary will be
offered an award.

®
* This corporate sponsor offers Special Scholarships in addition to Merit Scholarship awards, and Entry Forms that the sponsor
makes available must be filed.

PSAT/NMSQT Official Student Guide: National Merit® Scholarship Program 15


Corporate scholarship sponsors Solvay North America, LLC–4 awards for Finalists from
(continued) specified geographical areas who plan to pursue certain
■ college majors.

* Sony Electronics Inc.–3 awards for children of employees


SAP America, Inc.–Every Finalist who is the child of of Sony Electronics Inc. and Sony Corporation of America.
a U.S.-based employee of SAP and its entities will be
offered an award. South Broward Board of Realtors–2 awards for Finalists
from specified Florida high schools.
* Schindler Elevator Corporation–9 awards for children
of employees of the corporation. Southern Company Services, Inc.–Every Finalist who is
the child of an employee of Southern Company or a
* Schneider Electric/Square D Foundation–16 awards subsidiary will be offered an award.
for children of employees of Schneider Electric/Square D.
Southwest Airlines Co.–5 awards for Finalists who are
Schweinburg Fund–3 awards for Finalists from New children of employees of the company.
York City high schools.
Spirit AeroSystems, Inc.–7 awards for Finalists who
Science Applications International Corporation–10 are children of employees of the corporation and its
awards for Finalists who are children of employees of eligible subsidiaries.
SAIC and its subsidiaries.
* State Farm Companies Foundation–100 awards for
* Scripps Howard Foundation–5 awards for children of children of employees and insurance agents of State
employees of Scripps Howard and its subsidiaries. Farm Companies.
* Scripps Networks Interactive, Inc.–2 awards for chil- * Suburban Propane, L.P.–3 awards for children of
dren of employees of the corporation and its divisions employees of the company.
and subsidiaries.
Synopsys, Inc.–6 awards for Finalists who are children
* Sensient Technologies Foundation–3 awards for chil- of employees of the corporation and its subsidiaries.
dren of employees of Sensient Technologies Corporation
and its divisions and subsidiaries. ■
* Sentry Insurance Foundation, Inc.–5 awards for chil-
dren of employees of Sentry Insurance Group and its * Tate & Lyle Americas, Inc.–2 awards for children of
eligible subsidiaries. employees of the corporation and its subsidiaries.

The Shaw Group, Inc.–Every Finalist who is the child Taylor Publishing Company–1 award for a Finalist
of an employee of the corporation or a subsidiary will who is a member of a client yearbook staff.
be offered an award.
Telcordia Technologies–Every Finalist who is the child
* Siemens Foundation–60 awards for children of of an employee of Telcordia or a participating subsid-
employees of Siemens Corporation and its subsidiaries. iary will be offered an award.

* Snap-on Incorporated–5 awards for children of Tellabs Foundation–2 awards for Finalists who are
employees and dealers of the corporation. children of employees of Tellabs, Inc. and its wholly
owned subsidiaries.
Sodexo, Inc.–Every Finalist who is the child of an employee
of the corporation in the USA will be offered an award. * Teradata Corporation– 4 awards for children of
employees of the corporation.
Sogeti USA LLC–Every Finalist who is the child of an
employee of the corporation will be offered an award.

®
* This corporate sponsor offers Special Scholarships in addition to Merit Scholarship awards, and Entry Forms that the sponsor
makes available must be filed.

16 PSAT/NMSQT Official Student Guide: National Merit® Scholarship Program


Texas Instruments Incorporated–Every Finalist who Varian Medical Systems–2 awards for Finalists who are
is the child of an employee of the corporation will be children of employees of the corporation.
offered an award.
* Vulcan Materials Company Foundation–3 awards for
* Textron Charitable Trust–10 awards for children of children of employees of Vulcan Materials Company
employees of Textron Inc. and its subsidiaries. and its subsidiaries.
* J. Walter Thompson Company Fund, Inc.–2 awards
for children of employees of the company and its sub- ■
sidiaries.
Walgreen Co.–Every Finalist who is the child of an
* 3M Company–40 awards for children of employees of employee of the company will be offered an award.
the company and its affiliates and subsidiaries.
Waste Management–Every Finalist who is the child
* Tomkins Corporation Foundation–10 awards—5 of an employee of Waste Management Inc.’s family of
awards for children of salaried employees of Tomkins companies will be offered an award.
Corporation and Gates Corporation and their subsid-
iaries; 5 awards for children of hourly employees. * The Williams Companies Foundation, Inc.–10 awards
for children of employees of The Williams Companies,
Towers Watson and Company–1 award for the child of
Inc. and its subsidiaries.
an employee of the company.
* Wm. Wrigley Jr. Company–7 awards for children of
* The Travelers Employees Club–5 awards for children
employees of the company and its subsidiaries.
and specified relatives of members of The Travelers
Employees Club. * Wyeth–40 awards for children of employees of the cor-
poration and its subsidiary companies in the United
* Tredegar Corporation–2 awards for children of
employees of the corporation and its subsidiaries. States.

The Xerox Foundation–Every Finalist who is the child


■ of an employee of Xerox Corporation or a subsidiary
will be offered an award.
United Services Automobile Association–Every
Finalist who is the child of an employee of the com-
pany will be offered an award.

* United Space Alliance–20 awards for children of


employees of United Space Alliance.

* United States Fire Insurance Company–2 awards for


children of employees of Crum&Forster.

* The UPS Foundation–125 awards—100 awards for chil-


dren of full-time employees and 25 awards for children
of part-time employees of UPS and its subsidiaries.

* USG Foundation, Inc.–5 awards for children of employ-


ees of USG Corporation and its subsidiaries.

* Utility Workers Union of America, AFL-CIO–2 awards


for children of members of the union.

®
* This corporate sponsor offers Special Scholarships in addition to Merit Scholarship awards, and Entry Forms that the sponsor
makes available must be filed.

PSAT/NMSQT Official Student Guide: National Merit® Scholarship Program 17


College Merit Scholarship® sponsors.
The higher education institutions listed below currently are sponsors of National Merit Scholarships. The number in
parentheses reflects the minimum number of Merit Scholarship awards NMSC expects the college to offer annually.
However, neither the institution nor NMSC is obligated to offer these scholarships in 2011, 2012, or beyond.

■ Davidson College (3) Louisiana State University (25)


Denison University (3) Louisiana Tech University (3)
Abilene Christian University (3) DePauw University (10) Loyola University Chicago (3)
Albion College (3) Dickinson College (3) Luther College (3)
Alfred University (3) Drake University (3) Macalester College (35)
Alma College (3) Earlham College (3) Marquette University (3)
American University (10) Emory University (20) Messiah College (3)
Arizona State University, ■ Miami University (15)
all campuses (85) Michigan State University (35)
Auburn University (30) Florida State University (15) Michigan Technological University (5)
Austin College (3) Fordham University, all campuses (3) Mississippi State University (10)
Ball State University (3) Franklin and Marshall College (3) Missouri University of Science
Baylor University (35) and Technology (9)
Furman University (15)
Bethel University (Minnesota) (3) Montana State University-
George Washington University (15)
Birmingham-Southern College (10) Bozeman (3)
Georgia Institute of Technology (75)
Boise State University (3) New College of Florida (3)
Gonzaga University (3)
Boston College (7) New York University (90)
Gordon College (Massachusetts) (3)
Boston University (15) North Dakota State University (3)
Goshen College (3)
Bowdoin College (3) Northeastern University (3)
Grinnell College (15)
Bradley University (3) Northwestern University (50)
Gustavus Adolphus College (10)
Brandeis University (20) Hamline University (3) ■
Brigham Young University (10) Hampshire College (3)
Bucknell University (3) Harding University (10) Oberlin College (40)
Butler University (3) Harvey Mudd College (30) Occidental College (3)
■ Hendrix College (7) Ohio State University,
Hillsdale College (3) all campuses (90)
Hope College (6) Ohio University (6)
Calvin College (16)
Oklahoma City University (3)
Carleton College (75) ■
Oklahoma State University (10)
Case Western Reserve University (25)
Franklin W. Olin College of
Centre College (3) Illinois Wesleyan University (5) Engineering (3)
Claremont McKenna College (8) Indiana University Bloomington (6) Oregon State University (5)
Clemson University (3) Iowa State University (35) Ouachita Baptist University (3)
Colby College (Maine) (3) Ithaca College (3) Pennsylvania State University,
College of Charleston (3) Kalamazoo College (3) all campuses (5)
College of Wooster (3) Kansas State University (5) Pepperdine University (5)
Colorado College (6) Kenyon College (5) Pomona College (6)
Colorado State University (3) Knox College (3) Rensselaer Polytechnic Institute (15)
Concordia College (Minnesota) (3) Lawrence University (Wisconsin) (3) Rhodes College (11)
Creighton University (3) Lehigh University (3) Rice University (50)
Lewis & Clark College (5) Rochester Institute of Technology (7)
Liberty University (3) Rose-Hulman Institute of
Technology (15)

18 PSAT/NMSQT Official Student Guide: National Merit® Scholarship Program


Rutgers, The State University of University of Georgia University of Southern
New Jersey, all campuses (10) Foundation (20) California (35)
■ University of Houston (10) University of Southern
University of Idaho (8) Mississippi (5)
University of Illinois at Urbana- University of Tennessee (15)
Saint Louis University (3)
Champaign (50) University of Texas at Dallas (24)
St. Olaf College (20)
University of Iowa (20) University of Tulsa (55)
Samford University (5)
University of Kansas (28) University of Utah (20)
Santa Clara University (4)
University of Kentucky (10) University of Vermont (3)
Scripps College (3)
University of Louisville (12) University of Wisconsin-
South Dakota State University (3) Eau Claire (3)
University of Maine (3)
Southern Methodist University (15) University of Wisconsin-Madison (5)
University of Maryland,
Southwestern University (3) University of Wyoming (3)
all campuses (15)
Stony Brook University (3) Ursinus College (3)
University of Miami (15)
Tennessee Technological
University (3)
University of Minnesota, ■
all campuses (36)
Texas A&M University (125)
University of Mississippi (10)
Texas Christian University (3) Valparaiso University (5)
University of Missouri-
Texas Tech University (3) Vanderbilt University (18)
Columbia (20)
Transylvania University (3) Villanova University (5)
University of Montana (3)
Trinity University (12) Virginia Polytechnic Institute and
University of Nebraska-
Truman State University (3) Lincoln (20) State University (15)
Tufts University (10) University of Nevada, Las Vegas (3) Wabash College (3)
Tulane University (20) University of Nevada, Reno (3) Wake Forest University (4)
Washington and Lee University (15)
■ University of New Mexico (3)
University of North Carolina at Washington State University (3)
Chapel Hill (110) Washington University
University of Alabama at in St. Louis (50)
Birmingham (3) University of North Dakota (3)
University of North Texas (3) Wayne State University
University of Alabama at (Michigan) (10)
Tuscaloosa (30) University of Oklahoma,
all campuses (30) West Virginia University
University of Arizona (55) Foundation, Inc. (12)
University of Arkansas, University of Oregon (6)
Western Washington University (3)
Fayetteville (20) University of Pittsburgh (3)
Westminster College (Utah) (3)
University of Central Florida (7) University of Puget Sound (7)
Westmont College (3)
University of Chicago (60) University of Richmond (10)
Wheaton College (Illinois) (25)
University of Cincinnati (10) University of Rochester (20)
Whitman College (5)
University of Dallas (10) University of St. Thomas
(Minnesota) (3) Whitworth University (3)
University of Dayton (12)
University of the South (5) Willamette University (6)
University of Evansville (3)
University of South Carolina, Wofford College (3)
University of Florida (70)
all campuses (8) Worcester Polytechnic Institute (10)
University of South Florida (12) Xavier University (Ohio) (5)

PSAT/NMSQT Official Student Guide: National Merit® Scholarship Program 19


National Achievement ®
Scholarship Program
The National Achievement Scholarship Program was requesting consideration in the National Achievement
established in 1964 specifically to provide recognition for Program when they take the Preliminary SAT/National
outstanding Black American high school students and to
encourage them in their pursuit of higher education. This
Merit Scholarship Qualifying Test (PSAT/NMSQT ). The
two programs are conducted concurrently; however, a
®
academic competition is conducted by National Merit student’s standing in each program is determined indepen-
Scholarship Corporation (NMSC). dently. Black students can qualify for recognition, become
Black students who meet program entry requirements candidates for awards, and be honored as Scholars in both
may enter the National Achievement Program in addition competitions, but they can receive only one monetary
award from NMSC.
®
to the National Merit Scholarship Program by specifically

Steps in the 2012 National Achievement® Scholarship Competition


160,000 Entrants. In October 2010, Black Americans who take the PSAT/NMSQT®, complete Section 14 on
the answer sheet, and meet all other participation requirements will enter the 2012 competition for recognition and
National Achievement Scholarships. Nearly all participants (entrants) will be juniors planning to enter college in 2012.

High school principals will be asked to notify NMSC in the spring of 2011 of any
error in their students’ requests to enter the National Achievement Scholarship Program.

PARTICIPANTS WHO QUALIFY FOR NATIONAL ACHIEVEMENT PROGRAM RECOGNITION

3,100 Outstanding Partici- 1,600 Semifinalists. In early September 2011, participants


pants Referred to Colleges. who qualify as Semifinalists will be notified. They will receive National
In late September 2011, U.S. col- Achievement Scholarship Application materials explaining requirements
leges and universities will be sent a to advance in the competition for awards to be offered in 2012.
list of National Achievement Program
participants who have demonstrated
potential for academic success. These
Outstanding Participants will be sent 1,300 Finalists. In late January 2012, Semifinalists who advance
certificates through their schools, but to Finalist standing will be notified. All National Achievement Scholarship
they will not continue in the com- winners will be selected from this group of Finalists, based on informa-
petition for National Achievement tion submitted about their abilities, accomplishments, and goals.
Scholarships.

800 Achievement Scholarship ® winners. Beginning in


February 2012, the National Achievement Program will notify winners of:
• National Achievement® $2500 Scholarships and
• Corporate-sponsored scholarships
In early April, the names of Achievement Scholar® designees will be
released to news media for public announcement.

20 PSAT/NMSQT Official Student Guide: National Achievement® Scholarship Program


Program Entry through their high schools; however, these students will
not continue in the competition for National Achievement
Over 160,000 students currently enter each annual Scholarships.
National Achievement Scholarship Program. To partici-
pate in the 2012 competition, interested students must Semifinalists. Some 1,600 of the high scorers will be des-
take the 2010 PSAT/NMSQT, meet all requirements for ignated Semifinalists and have an opportunity to
entry to the National Merit Program listed on page 4, continue in the competition for National Achievement
and request consideration in the National Achievement Scholarships to be offered in 2012. Their names will be
Program by completing Section 14 of the test answer sent to regionally accredited four-year colleges and uni-
sheet. versities in the United States and released to news media
for public announcement.
Semifinalists in the National Achievement Program are
designated on a regional representation basis. The 50
states and the District of Columbia are grouped into sev-
eral geographic regions; an allocation of Semifinalists
is established for each that is proportional to the region’s
population of Black Americans. Semifinalists are the
highest scoring program participants in the states that
compose each region. Additional Semifinalists are named
Because the program is open only to Black American stu- in schools in U.S. commonwealths and territories, in U.S.
dents, high school officials are contacted before high boarding schools that enroll a sizable proportion of their
performing participants receive public recognition and students from outside the state in which the school is
asked to confirm their students’ participation requests. located, and in schools in other countries that enroll U.S.
The 2012 competition is explained in greater detail in citizens.
sections that follow. If you have a question that is not In the fall of 2011, NMSC will notify Semifinalists
answered in this Student Guide, talk with your counselor of their standing through their high schools. They will
or contact: receive National Achievement Scholarship Application
National Achievement Scholarship Program materials that explain requirements Semifinalists must
1560 Sherman Avenue, Suite 200 meet to advance in the competition.
Evanston, IL 60201-4897
Telephone: (847) 866-5100 Finalists. Some 1,300 Semifinalists in each year’s compe-
NMSC’s Web site address: www.nationalmerit.org tition advance to the Finalist level and receive a Certificate
of Achievement. Only Finalists are considered for National
Achievement Scholarships. To qualify as a Finalist, a
Program Recognition Semifinalist must:
Over 4,700 Black American high school students whose 1. continue to meet all requirements for participa-
performance on the 2010 PSAT/NMSQT places them tion in the National Achievement Program (including the
within the top 3 percent of program entrants will receive entry requirements listed on page 4);
recognition. Their Selection Index scores (the sum of crit- 2. be enrolled in the last year of high school and
ical reading, mathematics, and writing skills scores) will plan to enroll full time in college the following fall or be
be used to designate two groups of participants who will enrolled in the first year of college if grades 9 through 12
be honored by the 2012 National Achievement Program. were completed in three years or less;
Students referred to colleges. Approximately 3,100 of the 3. complete the National Achievement Scholarship
high-scoring program entrants will be recognized as Out- Application with all information requested, which includes
standing Participants in the 2012 National Achievement writing an essay;
Scholarship Program. In the fall of 2011, they will be
notified through their high schools that they are being 4. have a record of consistently high academic
referred to admission officials of regionally accredited performance in all of grades 9 through 12 and in any
four-year colleges and universities in the United States. college course work taken (the high school must provide
Certificates in recognition of their ability and potential a complete record of courses taken and grades earned by
for academic success in college will be sent to them the student, as well as information about the school’s cur-
riculum and grading system);

PSAT/NMSQT Official Student Guide: National Achievement® Scholarship Program 21


5. be fully endorsed for Finalist standing and
recommended for a National Achievement Scholarship by About 100 Achievement Scholarship awards will be
®
Corporate-sponsored Achievement Scholarship awards.

a high school official; offered in 2012 to Finalists who have qualifications that
particularly interest a corporate or business sponsor.
6. take the SAT and earn scores that confirm the
Almost all corporate-sponsored scholarships are desig-
PSAT/NMSQT performance that resulted in Semifinalist
nated for Finalists who plan to pursue college majors or
standing; and
careers the grantor wishes to encourage, are children of a
7. provide any other documentation and informa- sponsor’s employees, or residents of a company’s service
tion that NMSC requests. areas. Finalists who are considered for these scholarships
Before a Finalist can receive an award, he or she must are identified from information supplied on their schol-
notify the National Achievement Program of plans to arship applications, and winners are chosen from eligible
attend a regionally accredited college or university in the Finalists. In late February 2012, NMSC will mail scholar-
United States and enroll full time in an undergraduate ship offers to recipients.
course of study leading to a traditional baccalaureate Most corporate-sponsored Achievement Scholarship
degree. NMSC stipends are not payable for attendance at awards are renewable and provide stipends for up to four
service academies, virtual universities, and certain insti- years of undergraduate study, but some are nonrenewable
tutions that are limited in their purposes or training. awards that provide a single payment. (Values of these
awards are comparable to those in the National Merit
Program, described on page 7.)
Types of Awards and
Selection of Winners Achievement Scholarship®
In the 2012 competition, approximately 800 National
Achievement Scholarships worth more than $2.5 million
Sponsors
Currently, 42 corporations, company foundations, and
®
will be offered. All Achievement Scholar awardees will be
selected from the Finalist group, based on their abilities, other business organizations commit approximately
skills, and accomplishments. Although different types of $800,000 to support National Achievement Scholarships in
awards will be offered, no student can receive more than each annual competition. A sponsor may underwrite
one monetary award from NMSC. National Achievement $2500 Scholarships, for which
all Finalists compete, or scholarships for Finalists who
®
National Achievement $2500 Scholarships. Every
Finalist in the National Achievement Program will be
meet specified criteria; a few companies provide both
types of awards.
considered for one of these single-payment awards of
Sponsors and the number of Achievement Scholarship
$2,500. The 700 National Achievement $2500 Scholarships
awards they financed in 2010 are listed below. However,
will be awarded on a regional representation basis (as in
inclusion in a list does not commit any sponsor or NMSC
the naming of Semifinalists). Winners will be notified
to provide these Achievement Scholarship awards in
beginning in late February 2012.
2011, 2012, or beyond.
To judge which Finalists in a region have the greatest
potential for success in rigorous college studies, a com- ■
mittee of high school counselors and college admission
officers will evaluate the applications completed by all
Finalists and their school officials. Winner selections will ADP Foundation (2)
be based on Finalists’ academic records, demonstrated Ameren Corporation Charitable Trust (1)
leadership, contributions to school and community activ- Black Contractors United (1)
ities, test scores, essays, and school officials’ recommen-
dations. The Boule Foundation (12)
Companies and business organizations underwrite BP Foundation, Inc. (2)
some of the National Achievement $2500 Scholarships Chevron Corporation (3)
offered annually, but the majority of these awards are sup-
ported by NMSC’s own funds. Computer Sciences Corporation (1)
Con Edison (2)

22 PSAT/NMSQT Official Student Guide: National Achievement® Scholarship Program


■ Nationwide Foundation (1)
Navistar Foundation (1)
Delta Sigma Theta Sorority, Inc. (1) NewMarket Corporation (1)
Gaylord and Dorothy Donnelley Foundation (1) Novartis Corporation (1)
Ernst & Young Foundation (4) Occidental Petroleum Corporation (1)
ExxonMobil (2) Ortho-McNeil-Janssen Pharmaceuticals, Inc. (1)
Freeport-McMoRan Copper & Gold Foundation (1) PepsiCo Foundation, Inc. (1)
GEICO Philanthropic Foundation (1) PPG Industries Foundation (2)
General Mills Foundation (2)

GKN Foundation (2)
Goodrich Foundation, Inc. (1) Research Triangle Institute (1)
■ Reynolds American Foundation (5)
Sony Electronics Inc. (3)
The Links, Inc. (4) State Farm Companies Foundation (20)
The Links, Inc., Houston Chapter (1) United Services Automobile Association (1)
Loews Foundation (1) The UPS Foundation (7)
McDonald’s Corporation (1) Walgreen Co. (3)
The McGraw-Hill Companies, Inc. (4) The Williams Companies Foundation, Inc. (1)
Mead Witter Foundation, Inc. (4) The Xerox Foundation (12)
MetLife Foundation (3)
Motorola Foundation (3)

PSAT/NMSQT Official Student Guide: National Achievement® Scholarship Program 23


2010
Flip book over for
Scholarship
Competitions

conducted by
National Merit
Scholarship Corporation

Official
Student Guide
to the

Preliminary SAT/National Merit Scholarship Qualifying Test

Test-taking Help
• Seven Types of Questions
You’ll See on the Test
• Practice Test
With Answer Key
(Separate Pull-out Booklet)

www.collegeboard.com

82426-01442 • UNLWEB710

755929
Table of Contents The PSAT/NMSQT®
Contact information . . . . . . . . . . . . . . . . . . . . 2 The Preliminary SAT/National Merit Scholarship Qualifying Test (PSAT/NMSQT) is
cosponsored by the College Board and National Merit Scholarship Corporation. It is
My College QuickStart . . . . . . . . . . . . . . . . . . 3
developed and administered for the College Board and National Merit Scholarship
Things to know . . . . . . . . . . . . . . . . . . . . . . . . 3 Corporation by Educational Testing Service (ETS).
Important Information Contact the PSAT/NMSQT Office if you have suggestions, questions, or comments
about test registration, administration, or score reports. If you wish to withdraw your
Test scores. . . . . . . . . . . . . . . . . . . . . . . . . . . . . 4
answer sheet from scoring, or if you wish to report test administration irregularities,
Test regulations . . . . . . . . . . . . . . . . . . . . . . . . 4 you must notify the test supervisor or the PSAT/NMSQT Office immediately.
Special opportunities . . . . . . . . . . . . . . . . . . . 4 Write: PSAT/NMSQT Call: 609-771-7070
Student Search Service . . . . . . . . . . . . . . . . . . 5 P.O. Box 6720 609-882-4118 TTY
What to do if . . . . . . . . . . . . . . . . . . . . . . . . . . 5 Princeton, NJ 8 a.m. to 4 p.m. ET
you cannot take the test. . . . . . . . . . . . . 5 08541-6720 Fax: 610-290-8979
you are homeschooled . . . . . . . . . . . . . . 5 E-mail: [email protected]
you are studying in another country . . 5
you missed the test . . . . . . . . . . . . . . . . . 5
Students with disabilities . . . . . . . . . . . . . . . . 5 The College Board National Merit
Cancellation of scores. . . . . . . . . . . . . . . . . . . 6 The College Board is a not-for-profit Scholarship
Test fairness review . . . . . . . . . . . . . . . . . . . . . 6 membership association whose
mission is to connect students to
Corporation (NMSC)
Questioning a test question . . . . . . . . . . . . . . 6
college success and opportunity. NMSC is an independent, not-for-
College majors . . . . . . . . . . . . . . . . . . . . . . . . . 7 Founded in 1900, the College Board profit organization that operates
Prepare for the Test is composed of more than 5,700 without government assistance.
schools, colleges, universities and NMSC conducts two annual com-
Critical Reading Section . . . . . . . . . . . . . . . . 8 other educational organizations. petitions for recognition and
Critical Reading Skills Review . . . . . . . 8 Each year, the College Board serves
Critical Reading Questions . . . . . . . . . 9 seven million students and their
scholarships—the National Merit ®
Scholarship Program and the
Sentence completions . . . . . . . . . . . 9 parents, 23,000 high schools, and
3,800 colleges through major
®
National Achievement Scholarship
Passage-based reading . . . . . . . . . 10 Program. The PSAT/NMSQT is the
programs and services in college screening test for high school
Mathematics Section . . . . . . . . . . . . . . . . . . 17 readiness, college admission,
Mathematics Content Overview . . . . 17 students who wish to participate in
guidance, assessment, financial aid,
NMSC programs. Further
Calculator use . . . . . . . . . . . . . . . . . . . 17 and enrollment. Among its widely
information can be found in the
Mathematics Concept Review . . . . . . 18 recognized programs are the SAT , ® NMSC section on the flip side of
the PSAT/NMSQT and the
Mathematics Questions . . . . . . . . . . . 22 this publication or by visiting
Multiple-choice questions . . . . . . 22
Advanced Placement Program ® www.nationalmerit.org.
®
(AP ), SpringBoard and ® Please direct inquiries about any
Student-produced response ®
ACCUPLACER . The College
aspect of the National Merit
questions . . . . . . . . . . . . . . . . . . . . 26 Board is committed to the principles
of excellence and equity, and that Program or the National
Writing Skills Section . . . . . . . . . . . . . . . . . 29 Achievement Program, including
commitment is embodied in all of
Effective writing characteristics . . . . 29 its programs, services, activities and student participation requirements,
Improving sentences . . . . . . . . . . . . . . 30 concerns. For more information, the selection process and awards
Identifying sentence errors . . . . . . . . 31 visit www.collegeboard.com. offered, to:
Improving paragraphs. . . . . . . . . . . . . 32 The publication Guidelines on the
National Merit Scholarship
Uses of College Board Test Scores
Full-Length Practice Test Corporation
and Related Data, available at
1560 Sherman Avenue
(separate pull-out booklet inserted in center) www.collegeboard.com/research,
Suite 200
If you don’t have a practice test, ask your highlights proper and beneficial uses
of test scores and cautions against Evanston, IL 60201-4897
counselor for one. 847-866-5100
uses that are inappropriate.
The College Board
45 Columbus Avenue
New York, NY 10023-6992
212-713-8000

Reproduction of any portion of this Official Student Guide is prohibited without the written consent of the College Board.
© 2010 The College Board. All rights reserved. College Board, ACCUPLACER, Advanced Placement Program, AP, Student Search Service, SAT and the acorn logo are registered trademarks
of the College Board. My College QuickStart, and My SAT Study Plan are trademarks owned by the College Board. PSAT/NMSQT is a registered trademark of the College Board and National
Merit Scholarship Corporation. NMSC section pages 1–23 © 2010 National Merit Scholarship Corporation. The passages for sample questions have been adapted from published material. The
ideas contained in them do not necessarily represent the opinions of the College Board, National Merit Scholarship Corporation, or Educational Testing Service.

2 PSAT/NMSQT Official Student Guide: Prepare for the Test


My College QuickStart™ Things To Know
A Free Resource for Students about the
My College QuickStart is an online college and career planning
kit available free of charge to all students who take the PSAT/ PSAT/NMSQT
NMSQT. It is powered by your responses to the test, and pro-
vides personalized information that helps you take the next TEST DATES:
steps toward college: Wednesday, Oct. 13, or
Saturday, Oct. 16, 2010
◾ My Online Score Report—View your enhanced score report
with test questions, your answers, and the correct answers What does the PSAT/NMSQT measure?
with answer explanations. The PSAT/NMSQT measures critical reading,
math problem-solving and writing skills devel-


My SAT Study Plan —Prepare for the SAT with a person-
alized plan based on your test performance (includes a free
oped over many years, both in and out of school.
You will not have to recall facts from literature,
practice test). history, or science, or complete math formulas, as
◾ My Personality—Take a personality test to learn more about this test measures your reasoning and critical
yourself and find majors and careers that fit your strengths thinking skills.
and interests.
Why take the test?
◾ My Major & Career Matches—Learn about different majors ◾ Get feedback about critical academic skills
and careers and see what courses you should take now to be ◾ Prepare for the SAT
successful later. ◾ Start getting ready for college
◾ My College Matches—Get a starter list of colleges (based on ◾ Enter scholarship competitions
your home state and indicated choice of major) to help you What is the test fee?
begin your search. The fee for the 2010 PSAT/NMSQT is $13.
Schools sometimes charge an additional fee to
cover administrative costs. The College Board
makes fee waivers available to schools for juniors
from low-income families who can’t afford the test
fee. Talk to your counselor for more information
about fee waivers.
Whom can I contact if I have questions?
See your counselor. For further help, see contact
information on page 2 for the PSAT/NMSQT
Office, the College Board, and National Merit
Scholarship Corporation (NMSC).
What should I bring on test day?
◾ No. 2 pencils and eraser
◾ Approved calculator (see page 17)
◾ Social security number (optional)
◾ E-mail address (optional)
You may log in to My College QuickStart with the access code Online preparation and resources:
printed on your paper score report. The tool is available in Go to www.collegeboard.com/psat for sample
mid-December and can be utilized throughout high school. questions and practice test answer explanations.
Log in or take a tour at www.collegeboard.com/quickstart.

PSAT/NMSQT Official Student Guide: Prepare for the Test 3


Important Information
What scores will I receive? stored indefinitely, but personally identifying information
In December, your school will receive your PSAT/NMSQT will not be disclosed without your consent.
Score Report, which will be given to you, along with your Some school districts and states receive PSAT/NMSQT
test book, in the following weeks. Your score report will scores with other information about their students. In
include your critical reading, mathematics and writing addition, scores for students who qualify for programs
skills scores; score ranges; national percentiles; and described in the next section are reported to those
Selection Index (critical reading + mathematics + writing programs. The PSAT/NMSQT Program does not report
scores to colleges or commercial entities. Any businesses
skills scores), which NMSC uses as an initial screen of
that may contact you to sell their products or services
the large number of entrants to its scholarship competi-
did not receive student information from the College
tions. The score report will also show the correct Board as a result of taking the PSAT/NMSQT.
answers, the answers you gave and the difficulty level of
each question. If you have not received your score report Special Opportunities
by mid-January, see your counselor. The PSAT/NMSQT If you take the PSAT/NMSQT as a junior and qualify for
Program does not provide duplicate copies of score participation, the following programs may help you pre-
reports, but your school will have a copy. pare for college. If you do not want your name, scores
and other information released to them, write to the
Does anyone else receive my scores and informa- PSAT/NMSQT Office by Nov. 1, 2010.
tion I provide on my answer sheet?
PSAT/NMSQT Office
As cosponsors of the PSAT/NMSQT, the College Board
P.O. Box 6720
and NMSC receive the scores of students who take the
Princeton, NJ 08541-6720
test as well as information students provide on their
answer sheets. Some information is collected for very ►The National Hispanic Recognition Program (NHRP)
limited uses (noted on the answer sheet), and some is identifies outstanding Hispanic/Latino students and
used for research and to produce reports about groups shares information about them with interested colleges
of students. Scores and answer sheet information are and universities. To be eligible, you must be at least one-

Test Regulations
Standard rules and regulations give all students the same opportunity to do their best and prevent any student from having an
unfair advantage over other students.
► You must do all scratch work in the test player, wrist camera watch, pager, hand- ► If you become ill and/or must leave during
book and then mark your answers on the held organizer/computer), or any devices the test, or if for any other reason you do
answer sheet. You will not receive credit for that can access the Internet during the test not want your test scored, you may ask the
anything written in the test book. You may or the breaks. Power must be turned off test supervisor to destroy your answer sheet
not take your test book from the room. and these devices must be placed under before you leave the testing room. After you
► You may use an approved calculator (see your desk. If your watch has an alarm, you leave the testing room, any decision to with-
page 17) only during the mathematics sec- must turn that off as well. draw your answer sheet from scoring must
tions of the test; you may not have a calcu- ► You may not give or receive assistance or be reported immediately to the test supervi-
lator on your desk during the critical read- disturb others during the test or breaks. sor or the PSAT/NMSQT Office. (See con-
ing or writing skills sections and you may tact information on page 2.)
► You may check your work on the section
not share a calculator. on which you are being tested if you finish ► You may not discuss the contents of the test
► You are not allowed to use: protractors; com- before time is called, but you may not turn with anyone else, or share them through any
passes; rulers; dictionaries or other books; to any other section of the test. means, including but not limited to, e-mails,
pamphlets; papers of any kind; highlighters; text messages and the Internet until after
► You may take the PSAT/NMSQT only score reports have been distributed.
pens or colored pencils; listening, recording, once each school year. If you begin a test,
copying, or photographic devices; or any you are considered to have taken it. ► Members of your household or immediate
other aids. You may not bring food or drink family may not serve as PSAT/NMSQT
(including bottled water) into the test room, ► All PSAT/NMSQT test-takers in your supervisors or proctors, even at a different
unless preapproved for medical reasons. school must take the test at the same time. school, on the date that you take the test.
(Read about special arrangements and
► You may not use electronic or photographic testing accommodations for students with
devices (such as a cell phone, PDA, MP3 disabilities on page 5.) Date and sign on test day 10 13 10
Month Day Year

I agree to abide by the test regulations and certify that I am


the person whose name appears on this answer sheet.
What do I need to know about test regulations?
Read them carefully because you must sign a statement (shown on the right) on the test Shawn B. Studious
Signature
answer sheet stating that you will abide by these regulations. Also read about cancellation of
scores on page 6. Failure to follow these test regulations or any instructions given by the test Shawn B. Studious
supervisor may result in your scores being canceled. Name (Please print.)

4 PSAT/NMSQT Official Student Guide: Prepare for the Test


quarter Hispanic/Latino, as defined by the NHRP, and
meet a minimum PSAT/NMSQT cutoff score for your
What to do if...
state, as well as a minimum grade point average. ►you know in advance that you cannot take the
The College Board test on the date your school offers it:
National Hispanic Recognition Program Your school may be able to arrange for you to take the
11955 Democracy Drive test at a neighboring school that has selected the other
Reston, VA 20190 test date. Tell your counselor as soon as possible that
800-626-9795, ext. 5849 you have a conflict, such as not being able to test on
Saturday because of a religious observance. If you test at
►The National Scholarship Service (NSS) offers a free
another school, be sure to take your school’s code num-
college advisory and referral service for students who
ber and some form of photo identification with you.
plan to attend two-year or four-year colleges. Scores will
be sent for juniors who indicate that they are African ►you are homeschooled and want to take the
American or Black. PSAT/NMSQT:
You must make arrangements in advance with your local
National Scholarship Service
230 Peachtree Street, Suite 530 high school or other nearby school administering the test.
Atlanta, GA 30303 (For a listing of schools in your area, go to
404-522-7260 www.collegeboard.com/psat.) If this is not possible, con-
tact the PSAT/NMSQT Office.
►Telluride Association offers scholarships to highly
gifted juniors for summer seminars in the humanities ►you will be studying in another country when
and social sciences. the test is given:
You must make advance arrangements. Contact the
Telluride Association PSAT/NMSQT Office and provide the name of the city
217 West Avenue and country and, if known, the name and address of the
Ithaca, NY 14850 school you will be attending when the test is given. The
607-273-5011 PSAT/NMSQT Office will send you instructions.
What information about me will the College ►you missed the test but want to enter scholar-
Board Student Search Service® share? ship and recognition programs:
More than 1,700 colleges, universities, qualified scholar- For information about another route of entry to National
ship organizations, and nonprofit educational opportu- Merit Scholarship Corporation’s scholarship competi-
nity organizations use the Student Search Service to tions, see the box on page 5 of the NMSC section in the
connect with students. These institutions may select flip side of this Guide.
students within a specified range of scores, but they will
not receive your individual scores. Institutions may also
select students by other characteristics you provide, such Students with Disabilities
as sex, ethnic background, religious preference, citizen-
ship and self-reported GPA. The Student Search Service The PSAT/NMSQT Program provides testing accom-
will send these institutions only your name, address, sex, modations for students with documented disabilities. It
birth date, school, grade level, ethnic group, e-mail address is important to speak to your counselor or teacher right
and intended college major if you provide these items. Your away to determine what accommodations are necessary
information will only be released to educational organiza- to meet the needs of your disability. If you have not
tions, not commercial entities, and Student Search Service already been approved for accommodations by the
will not share your phone number with any organization. College Board Services for Students with Disabilities
You must be 13 years of age or older and in ninth through (SSD), you must submit a request for accommodations
twelfth grade to be included in Student Search Service. To (see your counselor). Please review the information in
preserve the integrity of the 7 Student Search Service ® the Instructions for Completing the 2010-2011 Student
information contained in the Some colleges, universities, scholar- Eligibility Form with your parent(s) or guardian(s) as
Student Search Service data- ship programs and nonprofit education
organizations may request information soon as possible. Use it to become familiar with the
you supply on this answer sheet (includ-
base, the College Board ing your e-mail address if you supply it)
in order to provide you with materials process and deadlines for requesting testing accommo-
reserves the right to remove, about educational opportunities and
financial aid. dations and to ensure that appropriate accommodations
with or without notice, a 7a. Would you like us to supply your are approved and arranged before test day.
particular student from the information for these purposes?

database. To participate, Yes No

7b. By providing your e-mail address in


answer “Yes” to the question the space below, you are granting
the College Board and NMSC per-
to the right on test day. mission to contact you via e-mail.

PSAT/NMSQT Official Student Guide: Prepare for the Test 5


After you have submitted an eligibility form, you will Test Fairness Review
receive a decision letter. If you are approved for accom-
modations, save this letter because it also indicates ETS develops and reviews tests by its own published fair-
your eligibility for accommodations on other College ness and quality assurance standards to ensure that the
Board tests, such as the SAT and Advanced Placement tests do not disadvantage examinees of a particular sex,
Program (AP) Exams. This eligibility will be in effect for race, ethnic group or cultural background. Following
as long as you remain in the same school and your these standards, ETS professional staff and external com-
school annually confirms your continued eligibility. If mittees, whose members are drawn from all regions of
you move to a new school, ask your counselor to notify the country, review all PSAT/NMSQT questions and
the College Board’s SSD office that you have moved. entire tests to eliminate any wording or content that
If you are interested in participating in NMSC’s schol- might be offensive to any examinees and to ensure that
arship programs and did not receive accommodations the test includes references to men and women, and to
for which you were approved or did not receive approval individuals from a variety of racial, ethnic, and cultural
in time for the test, please contact NMSC immediately backgrounds. To further ensure fairness, ETS performs a
but no later than Nov. 15. statistical analysis, followed by professional review, which
reduces the possibility that any question might be more
Cancellation of Scores difficult for any particular group of students than would
be expected from their performance on the total test.
To report scores that accurately reflect performance, Differences in average test performance of various groups
ETS maintains, on behalf of the College Board and can be attributed to many factors, such as long-term edu-
NMSC, test administration and security standards cational preparation; the test itself reflects such differences
designed to give all students the same opportunity to but is not the cause.
demonstrate their abilities and to prevent any student
from gaining an unfair advantage because of testing Questioning a
irregularities or misconduct. ETS reviews irregularities
and test scores believed to have been earned under Test Question
unusual circumstances.
Students may be dismissed from the testing room and If you find what you consider to be an error or ambiguity
their answer sheets may be destroyed if they fail to follow in a test question, tell the test supervisor immediately
test regulations or instructions given by the test supervi- after the test. You may also write to: Assessment
sor. Once answer sheets are submitted for scoring, ETS Development, P.O. Box 6656, Princeton, NJ 08541-6656;
reserves the right not to score the answer sheet of a stu- or send an e-mail to: [email protected].
dent who engaged in misconduct or was involved in a State in your inquiry your name and mailing address,
testing irregularity. the date you took the PSAT/NMSQT, the name and
ETS reserves the right to cancel scores if there is address of the school where you took the test, the test
reason to doubt their validity. Before acting, ETS will section, the test question (as well as you can remember)
inform the student of the reasons for questioning the and an explanation of your concern about the question.
scores and will give the student an opportunity to pro- ETS will send you a written response after your
vide additional information, to confirm the scores inquiry has been reviewed thoroughly by subject-matter
by taking another PSAT/NMSQT, or to cancel the scores. specialists. (Note: ETS will not respond via e-mail, so be
The student may also request arbitration in accordance sure to include your full name and mailing address.)
with ETS’s Standard Arbitration Agreement. If the response does not resolve your concern, you can
Score reviews are confidential. If it is necessary to request that the Director of Assessment Development
cancel reported scores, ETS will notify score users, but initiate further reviews of your inquiry. If you still have
the reasons for cancellation will not be disclosed. This concerns, you can request a formal review of your
policy does not necessarily apply in group cases. inquiry by an independent review panel.
ETS will forward all reports of misconduct or testing ETS will retain your letter (with your identification
irregularities to NMSC, which reserves the right to make deleted) along with other such letters. You can obtain
its own independent judgment about a student’s partici- copies of these letters for a nominal fee by writing to
pation in its scholarship programs. Assessment Development.

6 PSAT/NMSQT Official Student Guide: Prepare for the Test


College Majors My College QuickStart account will let you investigate
this and other majors further.
When you take the PSAT/NMSQT, indicate the college If you say “Yes” to Student Search Service, colleges and
major that best matches your interests by entering the universities that offer degrees in your area of interest can
code from the list below on your answer sheet. Your send you information about their programs.

College Major Codes


Agriculture, Agriculture Operations Computer and Information Linguistics - 559 Wildlife, Fish, Wildlands Science/
and Related Sciences - 100 Sciences and Support Russian Language and Management - 796
Agricultural Business and Services - 300 Literature - 560
Management - 101 Artificial Intelligence - 301 Spanish Language and Parks, Recreation, Leisure and
Animal Sciences - 103 Computer Programming - 302 Literature - 561 Fitness Studies - 800
Computer Science - 303 Parks and Recreation Facilities
Architecture and Related Computer Systems Networking Health Professions and Related Management - 802
Services - 120 and Telecommunications - 306 Programs - 600 Sport and Fitness Administration/
Architecture - 121 Information Sciences/Studies - 308 Allied Health Diagnostic, Management - 803
City/Urban, Community and Intervention and Treatment
Regional Planning - 123 Education - 400 Professions - 602 Personal and Culinary
Landscape Architecture - 125 Early Childhood Education and Athletic Training/Trainer - 603 Services - 810
Teaching - 407 Clinical/Medical Laboratory
Area, Ethnic, Cultural, Gender Elementary Education and Science/Research and Allied Philosophy and Religious
and Group Studies - 140 Teaching - 417 Professions - 605 Studies - 820
Area Studies - 141 Secondary Education and Communication Sciences and Philosophy - 821
Ethnic, Cultural Minority, Gender Teaching - 430 Disorders - 606 Religion/Religious Studies - 822
and Group Studies - 142 Special Education and Dental Hygiene/Hygienist - 607 Physical Sciences - 830
Teaching - 432 Dietetics and Clinical Nutrition Astronomy - 832
Biological and Biomedical Services - 610
Sciences - 160 Engineering - 450 Astrophysics - 833
Nursing - 619 Atmospheric Sciences and
Biology/Biological Sciences, Aerospace, Aeronautical and Predentistry Studies - 627
General - 161 Space Engineering - 451 Meteorology - 834
Premedicine/Premedical Chemistry - 836
Biochemistry - 162 Agricultural Engineering - 452 Studies - 628
Biophysics - 163 Architectural Engineering - 453 Geological and Earth Sciences/
Preoccupational Therapy Geosciences - 837
Biotechnology -164 Bioengineering and Biomedical Studies - 678
Cell/Cellular Biology and Anatomical Engineering - 454 Materials Science - 854
Prepharmacy Studies - 629 Physics - 843
Sciences - 166 Chemical Engineering - 455 Prephysical Therapy Studies - 682
Ecology - 167 Civil Engineering - 456 Preveterinary Studies - 630
Genetics - 168 Psychology - 870
Computer Engineering,
Marine Biology and Biological General - 457 History - 700 Public Administration and Social
Oceanography - 169 Electrical and Electronics Services - 880
Microbiological Sciences and Engineering - 459 Homeland Security, Law Human Services, General - 881
Immunology - 170 Engineering Physics/Applied Enforcement, Firefighting Public Administration - 882
Molecular Biology - 171 Physics - 461 and Related Protective Public Policy Analysis - 883
Neuroscience - 185 Engineering Science - 462 Services - 890 Social Work - 884
Zoology/Animal Biology - 175 Environmental/Environmental Criminal Justice/Law Enforcement
Health Engineering - 463 Administration - 892 Social Sciences - 900
Business Management, Geological/Geophysical Forensic Science and Anthropology - 901
Marketing and Related Engineering - 464 Technology - 894 Archaeology - 902
Support Services - 200 Industrial Engineering - 465 Homeland Security - 897 Criminology - 903
Accounting and Related Materials Engineering - 466 Economics - 904
Services - 201 Mechanical Engineering - 467 Legal Professions and Geography - 905
Actuarial Science -202 Mining and Mineral Engineering - 468 Studies - 710 International Relations and
Business Administration, Nuclear Engineering - 470 Prelaw Studies - 712 Affairs - 906
Management and Petroleum Engineering - 472 Political Science and
Operations - 204 Liberal Arts and Sciences,
Polymer/Plastics Engineering - 475 General Studies and Government - 907
Fashion Merchandising - 208 Sociology - 908
Finance and Financial Management Engineering Technologies - 500 Humanities - 720
Services - 209 Computer Engineering Humanities/Humanistic Theology and Religious
Hospitality Administration/ Technology - 501 Studies - 722 Vocations - 920
Management - 211 Drafting/Design Engineering Liberal Arts and Sciences/Liberal
Hotel, Motel and Restaurant Technologies - 502 Studies - 723 Visual and Performing Arts - 940
Management - 225 Telecommunications Technology/ Art History, Criticism and
Human Resources Management Mathematics and Statistics - 740 Conservation - 941
Technician - 509 Applied Mathematics - 741
and Services - 212 Arts, Entertainment and Media
International Business - 214 English Language and Literature/ Mathematics - 742 Management - 960
Management Information Systems Letters - 520 Statistics - 743 Dance - 942
and Services - 217 Creative Writing - 522 Drama and Theater Arts - 943
Marketing/Marketing Military Technologies and Applied Fashion/Apparel Design - 945
Management - 218 Family and Consumer Sciences/ Sciences - 360 Film/Video and Photographic
Meeting and Event Planning - 230 Human Sciences - 540 Arts - 946
Foods, Nutrition and Wellness Multi/Interdisciplinary
Studies - 770 Fine and Studio Art - 947
Communication, Journalism and Studies - 542 Game and Interactive Media
Related Programs - 250 Biological and Physical
Sciences - 771 Design - 970
Communication and Media Foreign Languages, Literatures Graphic Design - 948
Studies - 252 and Linguistics - 550 International/Global Studies - 772
Mathematics and Computer Interior Design - 949
Digital Communication and Media/ Classics and Classical Music - 950
Multimedia - 253 Languages, Literatures and Science - 773
Photography - 954
Journalism - 254 Linguistics - 553 Technical Theater/Theater Design
Comparative Literature - 554 Natural Resources and
Public Relations, Advertising and Conservation - 790 and Technology - 955
Applied Communication - 270 East Asian Languages, Literatures
and Linguistics - 555 Environmental Science - 791
Radio and Television - 257 Environmental Studies - 792 OTHER - 990
French Language and
Literature - 556 Fishing and Fisheries Sciences and UNDECIDED - 999
German Language and Management - 793
Literature - 557 Forestry - 794

PSAT/NMSQT Official Student Guide: Prepare for the Test 7


Prepare for the Test • SAMPLE TEST QUESTIONS
• TEST DIRECTIONS
• TEST-TAKING TIPS

Critical Reading Section Comprehending Sentences


Sentences are composed of one or more clauses that
together create meaning. The relationships between the
Critical Reading Skills Review elements of a sentence are usually controlled by the
Become familiar with the following reading skills before following:
test day. For additional test-taking tips, visit ◾ Punctuation
www.collegeboard.com/psat.
A colon is often used to introduce material that
Building a Vocabulary explains or illustrates what has come before. Example:
The best way to learn the meanings of words is to read “Last night, John displayed dreadful manners: he
widely and to look up unfamiliar words in a dictionary. arrived late for dinner, criticized his host’s appearance
When a dictionary is not available, you can try the and refused to help with the dishes.”
following:
A semicolon is often used to join equal and balanced
◾ Determine meaning from word components. sentence elements. When it joins two independent
A prefix appears at the front of a word and affects the clauses, it indicates a closer relationship between the
meaning of what follows. Mal, for example, means clauses than a period does. Example: “Rodriguez’s new
bad, so words beginning with mal (such as novel is brilliantly written; surprisingly, it has sold
malpractice, maladjusted, malodorous) usually have a poorly.”
negative meaning. There are many common prefixes,
◾ Introductory and transitional words and phrases
including ante, dis, non, post and un.
Some words and phrases connect similar ideas: also,
A suffix appears at the end of a word and affects the
and, for example, furthermore, in addition, in other
meaning of what comes before. Less, for example,
words, likewise, moreover.
means lacking, so words ending in less (such as
clueless, guileless, worthless) usually involve being Other words and phrases connect ideas that are
without something. Common suffixes include able, dissimilar in meaning: although, but, despite, except,
ism, ology and tion. however, in contrast, instead, nevertheless, nor, not, on
the other hand, yet.
A root is the element of a word that carries the main
component of meaning. There are many common Some words and phrases connect ideas in cause-and-
roots in the English language. Aqua, for example, effect relationships: as a result, because, consequently,
means water and is the root in words such as hence, in order to, therefore, thus.
aquarium and aquatic. Chrom means color and is the Other words indicate that a certain condition must be
root in words such as chromatic and monochrome. considered: if, when.
When you come across an unfamiliar word, you can
sometimes figure out its meaning if you recognize a Reading Effectively
root from a more familiar word. A good reader is an active reader, one who moves
beyond what is literally stated and draws inferences
◾ Determine meaning from context. about what he or she reads. To be an active reader, you
You can often figure out the approximate meaning of should be able to do the following:
a word from its context. If, for example, you ◾ Understand the primary purpose or main idea of a
encountered the sentence “Mary was a gregarious piece.
person and therefore got to know many people,” you
could probably guess that “gregarious” means sociable. Is the author seeking to entertain, to inform, or to
But, whenever you can do so, you should confirm convince? Try to distinguish between the main idea and
the meaning of unfamiliar words by checking a supporting ideas.
dictionary.

8 PSAT/NMSQT Official Student Guide: Prepare for the Test


◾ Understand the tone or attitude conveyed by certain Sentence Completions
words and expressions. Sentence completion questions measure your
Do you sense that the author is critical or enthusiastic? knowledge of the meanings of words and your ability
Earnest or humorous? Pay close attention to the to understand how the different parts of a sentence fit
connotations of key words and what they may say together logically. The sentences, usually adapted from
about the author’s attitude. Become familiar with terms published material, cover a wide variety of topics of the
that characterize tones—words like caustic, didactic, sort you are likely to have encountered in school or in
irreverent, pensive and satirical. your general reading. Your understanding of sentences
will not depend on specialized knowledge of science,
◾ Understand the use of rhetorical strategies. literature, social studies or any other field.
Note the techniques by which writers achieve their Below are the directions you will see on the test.
effects. Does the author use examples, figurative
language, imagery, irony, overstatement, quotations, Each sentence below has one or two blanks, each blank
rhetorical questions or word repetition? Ask yourself indicating that something has been omitted. Beneath
why the author chose to express things a certain way. the sentence are five words or sets of words labeled A
through E. Choose the word or set of words that, when
◾ Recognize implications and make evaluations. inserted in the sentence, best fits the meaning of the
When you infer, you go beyond what is literally stated; sentence as a whole.
you piece together what is implied by certain words, Example:
phrases, and statements. This may involve recognizing
underlying assumptions, understanding how different Hoping to ------- the dispute, negotiators proposed
ideas relate to one another or evaluating the limits of a compromise that they felt would be ------- to both
an argument. labor and management.
(A) enforce . . useful
Critical Reading Questions (B) end . . divisive
(C) overcome . . unattractive
The critical reading section of the PSAT/NMSQT (D) extend . . satisfactory
contains two types of questions: (E) resolve . . acceptable A B C D E

Sentence Completion (13 questions)


Passage-Based Reading (35 questions) To answer a sentence completion question, you have to
Approaches to the Critical Reading Section understand how the parts of the sentence relate to one
◾ Work on sentence completion questions first. They another. In the preceding example, the introductory
take less time to answer than the passage-based clause (before the comma) indicates what the negotiators
reading questions. are hoping to do.
◾ The difficulty of sentence completion questions ◾ Which of the first terms makes sense when inserted
increases as you move through a question set. in the first blank? If the negotiators are proposing a
◾ Reading questions do not increase in difficulty. “compromise,” then they are probably seeking to end,
Instead, they follow the logic of the passage. overcome or resolve the dispute. Choices (B), (C),
◾ The information you need to answer each reading and (E) all seem reasonable so far.
◾ What about the second blank? If the negotiators are
question is in the passage(s). Reading carefully is
the key to finding the correct answer. Don’t be seeking to end, overcome or resolve the dispute,
then it’s likely that they would propose a compromise
misled by an answer that looks correct but is not
suitable to both labor and management. The second
supported by the actual text of the passage(s).
terms in choices (A), (D) and (E) could all help to
◾ Do not jump from passage to passage. Stay with a
make this point.
passage until you have answered as many questions
◾ So which is the best answer? Choice (E), because both
as you can. When you have gone through all the
words in this choice work together to complete the
questions associated with a passage, go back and logic of the sentence: “Hoping to resolve the dispute,
review any you left out or were not sure about. negotiators proposed a compromise that they felt
◾ In your test book, mark each question you don’t would be acceptable to both labor and management.”
understand so that you can easily go back to it later
if you have time. Tip: Try answering two-blank questions one blank at a
time. If you can eliminate one word in an answer, the
entire choice can be eliminated.

PSAT/NMSQT Official Student Guide: Prepare for the Test 9


1. Before Karen Chin’s research, scientists assumed that the 2. Greta praised the novel for its -------, claiming it depicted
value of evidence preserved in the fossils called coprolites reality so vividly that it seemed more like fact than
was too ------- to warrant the effort of -------. fiction.

(A) unpredictable . . transformation (A) transcendence (B) romanticism


(B) superlative . . examination (C) impenetrability (D) loquacity
(C) conventional . . eradication (E) verisimilitude
(D) relevant . . synthesis
(E) dubious . . analysis The material in the second part of this sentence helps
define the word that best completes the blank.
The sentence indicates what scientists had thought Verisimilitude, choice (E), is the quality of appearing to
about the value of evidence preserved in coprolites be true or real. If Greta claims that the novel depicts
before Karen Chin’s research: they had assumed that reality “so vividly” that it seems “more like fact than
it “was too ------ to warrant the effort of -------.” The fiction,” then she would be praising the novel for its
word that best completes the first blank will characterize verisimilitude. While the words in the other options are
the scientists’ view of the evidence, and the word that sometimes used to describe novels, none of them makes
best completes the second blank will indicate what the sense when inserted in this particular sentence.
scientists therefore thought was not worth the effort of ◾ Choice (A), transcendence, would suggest that the
doing. Choice (E) is the correct response. If the scientists novel rises beyond the scope of ordinary experience,
thought the value of the evidence preserved in coprolites but nothing in the sentence indicates that it does this.
was dubious, or doubtful, then they probably would not ◾ Choice (B), romanticism, would suggest that the
think it worth the effort of analysis.
novel focuses on the heroic or the adventurous. But
◾ Choice (A) is incorrect. While unpredictable may
Greta praises the novel for its realism, not for its
make some sense in the first blank, transformation
romantic qualities.
does not make sense in the second. One would not
◾ Choice (C), impenetrability, would suggest that the
expect scientists to make an effort to transform, or
novel is difficult or impossible to understand. If the
change, the evidence.
novel were impenetrable, Greta would be unlikely to
◾ Choice (B) is incorrect. If scientists thought the value
praise it.
of the evidence preserved in coprolites was super-
◾ Choice (D), loquacity, would suggest that the novel is
lative, or of very high quality, then it probably would
full of excessive talk, but nothing in the sentence
“warrant the effort of examination.”
indicates this.
◾ Choice (C) is incorrect. For one thing, scientists
would not be expected to attempt the eradication, or Tip: Do not choose a word because it sounds good in
the complete elimination, of evidence. And it would one part of the sentence. Pick the answer choice that best
be especially unlikely for scientists to feel that some completes the logic of the entire sentence.
evidence was “too conventional,” or too customary, to
“warrant the effort” of such eradication. Passage-Based Reading
◾ Choice (D) is incorrect because it would be illogical to
Many questions in the critical reading section of the
speak of evidence as being “too relevant” to be worth PSAT/NMSQT will measure your ability to read,
the effort of synthesis. understand and interpret reading passages. These
passages are drawn from a variety of fields, including the
humanities, social studies and natural sciences. Passages
may also be taken from works of fiction. The selections
will vary in style and may include narrative,
argumentative and expository elements. Passages are
usually drawn from actual published works, though this
material is often adapted for testing purposes.
Passages will range in length from about 100 to 850
words and will often include an introduction and/or
footnotes. Some questions will be based on a pair of
passages on a shared theme or issue.
The directions you will see on the test are at the top of
the next page.
10 PSAT/NMSQT Official Student Guide: Prepare for the Test
The passages below are followed by questions based on their content; questions following a pair of related
passages may also be based on the relationship between the paired passages. Answer the questions on the basis
of what is stated or implied in the passages and in any introductory material that may be provided.

Question 3 is based on the following passage. Questions 4–5 are based on the following passages.
It may look as though I do not know how to begin Passage 1
my tale. Funny sight, the elderly gentleman who comes
I believe that all forms of popular culture—rock and
lumbering by in a valiant dash for the bus, which he
rap music, sci-fi and horror films, cartoons and comic
Line eventually overtakes but is afraid to board in motion and
strips, etc.—should be banned from college courses in
(5) so, with a sheepish smile, drops back, still going at a trot.
Line the arts and the humanities. Typically today students
Is it that I dare not make the leap? It roars, gathers
(5) arrive on college campuses already besotted with the trash
speed, will presently vanish irrevocably around the corner,
of popular culture, and it must now be one of the goals of
the bus, the motorbus, the mighty motorbus that is my
a sound liberal education to wean them away from it—or,
tale. Rather bulky imagery, this. I am still running.
if that is asking too much (I don’t think it is, but if that
really is too much), then at least to educate them to per-
3. In context, the actions of the “elderly gentleman” (line 2) (10) ceive what the differences are between high culture and
are best understood as the trash that impinges on so much of their leisure time.
Passage 2
(A) a sign that the narrator fears he will miss an
Although there are legions of crabs, cranks, and cur-
appointment
mudgeons who proclaim that all popular culture is worth-
(B) a reference to the narrator’s physical skills
less garbage and/or responsible for crime, short attention
(C) a symbol of the narrator’s social status (15) spans, and disrespect for elders, nobody who knows any-
(D) a metaphor for the narrator’s literary struggles thing about popular culture has so simple a relationship to
(E) an allusion to the narrator’s artistic reputation the stuff. Nobody says, “I just love all movies,” or “I like
pretty much every song I hear.” On the contrary, devel-
To answer this question successfully, you must make the oping the faculty of discrimination is part of the fun of
connection between the “elderly gentleman” trying to (20) immersing oneself in the popular—which means, inter-
catch the bus and the narrator trying to begin his story. estingly, that few fans of popular culture are wholly
In the first sentence, the narrator says he does not know “immersed” in it. To be a really knowledgeable fan, in
“how to begin” his tale, and in the next sentence, he other words, you usually have to be a keen critic. It is
describes an elderly gentleman who is “afraid to board” a the people who cannot stand pop culture who are truly
(25) indiscriminate.
moving bus. The narrator then asks, “Is it that I dare not
make the leap?” This question invites the reader to see
the similarity between the gentleman’s fear of boarding 4. Which best describes the respective attitudes of the
the bus in motion and the narrator’s difficulty beginning author of Passage 1 and the author of Passage 2 toward
his story. The subsequent sentence, in which the nar- popular culture?
rator refers to “the mighty motorbus that is my tale,”
makes the metaphor explicit. The actions of the gentle- (A) Scathing versus regretful
man are therefore best understood as “a metaphor for (B) Dismissive versus receptive
the narrator’s literary struggles,” making choice (D) the (C) Impartial versus appreciative
best response. (D) Arrogant versus ambivalent
◾ Choice (A) is incorrect because there is no suggestion (E) Judgmental versus uninterested
in the passage that the narrator is trying to make “an
appointment.” This type of question asks you to understand the overall
◾ Choice (B) is incorrect because the passage makes no
attitude expressed in each passage toward a topic or
reference to the narrator’s “physical skills.” Rather, it issue—in this case, popular culture. You’ll note that the
represents his literary difficulties. author of Passage 1 twice refers to popular culture as
◾ Choice (C) is incorrect because the narrator’s “social “trash” and argues that it should be banned from college
status” is not at issue in the passage. courses. Such an attitude can best be described as
◾ Choice (E) is incorrect because it is not clear whether “dismissive.” The author of Passage 2, on the other hand,
the narrator even has an “artistic reputation.” At the says that immersing oneself in popular culture can help
end of the passage, he is “still running,” still trying to one develop the “faculty of discrimination,” that
begin his tale.
PSAT/NMSQT Official Student Guide: Prepare for the Test 11
becoming a knowledgeable fan involves becoming a ◾ Choice (A) is incorrect because the characterization in
“keen critic.” This author can be described as “receptive” Passage 1 cannot be called “forgiving.”
to popular culture, or open to the idea that things can be ◾ Choice (B) is incorrect. While the author of Passage 1
learned from it. Choice (B) is therefore the best may indeed be “argumentative,” nothing in Passage 2
response. Remember that both parts of a choice must be suggests that students regard popular culture as
accurate for it to be considered the correct answer. “passé.”
◾ With choice (A), the author of Passage 1 is indeed ◾ Choice (C) is incorrect because Passage 2 doesn’t
“scathing,” but the author of Passage 2 is not suggest that some students are “unfamiliar with
“regretful.” popular culture.”
◾ With choice (C), the author of Passage 2 is ◾ Choice (E) is incorrect. The author of Passage 2 might
“appreciative,” but the author of Passage 1 is not at all find the characterization of the students in Passage 1
“impartial.” too “harsh,” but not because they have not yet begun
◾ With choice (D), some might consider the author of their education.
Passage 1 to be “arrogant,” but the author of Passage 2
is not “ambivalent.” Questions 6–8 are based on the following passage.
◾ With choice (E), the author of Passage 1 might be
considered “judgmental,” but the author of Passage 2 This passage is adapted from a 1987 essay by a physicist
who was inspired by watching a snowstorm.
is certainly not “uninterested.”
On New Year’s Day in 1611, the astronomer Johannes
Tip: When answering questions about a pair of passages, Kepler presented his patron John Wacker, Counsellor to
be sure to distinguish what one author says from what the the Imperial Court, a little book entitled The Six-Cornered
other one says. Line Snowflake. It was also the first recorded step toward a
(5) mathematical theory of natural form.
5. The author of Passage 2 would most likely argue that the Why, asks Kepler in his little treatise, do snowflakes fall
characterization of the “students” in Passage 1 is too as six-cornered starlets, “tufted like feathers”? There must
be a cause, he asserts, for if it happens by chance, then
(A) forgiving, because consumers should be held more why don’t snowflakes fall with five corners or with seven?
accountable for their tastes (10) Casting about for an answer, Kepler considers other
(B) argumentative, because many students now regard hexagons in nature: the shape of the cell in a honeycomb,
popular culture as passé for example. He shows that a hexagonal architecture for
(C) simplistic, because it fails to acknowledge that the honeycomb exactly suits the bee’s purpose, for (as
some students are in fact unfamiliar with Kepler proves) the hexagon is the geometrical figure that
popular culture
(15) enables the bee to enclose a maximum volume of honey
with a minimum of wax. Next Kepler considers the seeds
(D) sweeping, because many consumers of popular
of the pomegranate, which are also hexagonal in form.
culture are actually quite discerning in their
He demonstrates that this is the shape any round, pliable
judgments
object will take if a mass of such objects is squeezed equally
(E) harsh, because these students have yet to begin their (20) from every side into a minimal volume, as the seeds of
education the pomegranate are squeezed together in the growing
fruit. Then Kepler reviews other possible “causes” for the
In lines 5–6, the author of Passage 1 claims that students snowflake’s six-sided elegance: formal causes, efficient
today are “besotted with the trash of popular culture,” causes. He considers the role of beauty, function, and
suggesting that they are infatuated with it or stupefied by (25) necessity. Perhaps, Kepler muses somewhat whimsically,
it. In lines 18–25, the author of Passage 2 takes a more snowflakes take care “not to fall in an ugly or immodest
charitable view, arguing that many fans of popular fashion.” Or maybe, he concludes, in making snowflakes,
culture are in fact quite “knowledgeable” and that they nature simply “plays.”
have developed their “faculty of discrimination.” Since By inverting my pocket binoculars, I can magnify the
the author of Passage 2 believes that many consumers of (30) snowflakes on my sleeve. It is easy to see why Kepler
could imagine that nature “plays.” The snowflakes are
popular culture are quite discerning, this author would
like the patterns in a child’s kaleidoscope. But Kepler
likely view the characterization of the “besotted” knew that “play” cannot be the entire story. At the end
students in Passage 1 as too “sweeping,” because it fails of his little book, Kepler confesses his ignorance and
to make appropriate distinctions. Choice (D) is therefore (35) leaves the problem of the snowflake’s symmetry to future
the best answer. generations of natural philosophers.

12 PSAT/NMSQT Official Student Guide: Prepare for the Test


The riddle of the snowflake has since been partly 7. In lines 28 and 31, “plays” most nearly means
solved. Physicists have traced the snowflake’s six-sided
secret down into the heart of matter, to the form of (A) competes
(40) the water molecule, and, ultimately, to the laws of (B) mimics
atomic bonding that give the water molecule its shape. (C) pretends
Kepler would be pleased to know that the beauty of the (D) wagers
snowflake is founded upon principles of mathematical (E) frolics
order. But he would have been surprised to learn that
(45) atoms play a role in the explanation. Kepler rejected Read the sentences in which the quoted word appears
atomism because he assumed that a rattling, clattering and try to determine how that word is being used in
chaos of atoms could never give rise to the elegant context. Then pick the answer choice that comes closest
symmetries of nature. But if twentieth-century physics
to this meaning. To “frolic” is to amuse oneself, to play
has taught us anything, it is that nature accommodates
(50) beautiful form even at the level of the atom. Beauty in in a light-spirited fashion. Kepler suggests that in
nature is not something that shows up only at a certain making snowflakes, nature may just be frolicking.
level of complexity. Beauty is built into every jot and Choice (E), “frolics,” is the correct answer.
tittle of creation—into every atomic brick! Beauty soaks ◾ Choice (A), “competes,” is a possible meaning of
reality as water fills a rag.
“plays,” but one that makes little sense in this context.
It is hard to see how nature could “compete” by
6. The first paragraph (lines 1–5) primarily functions to making snowflakes.
◾ Choice (B), “mimics,” is incorrect because Kepler is
(A) formulate a generalization not suggesting that nature is imitating anything when
(B) define a problem
making snowflakes.
(C) underscore a scientist’s notoriety
◾ Choice (C), “pretends,” is a possible meaning of
(D) provide a historical context
(E) question a prevalent hypothesis
“plays,” but one that makes no sense in this context.
◾ Choice (D), “wagers,” is a meaning of “plays,” but one
To answer this question correctly, you must recognize that makes no sense in this context.
how the first paragraph functions in the passage. Since Tip: For this type of question, do not just pick a synonym
this paragraph offers background information on for the quoted word. Pick the answer choice with the
Kepler’s book, indicating when and for whom it was meaning that is closest to the way the quoted word is used in
written and hinting at its mathematical and scientific the passage.
importance, it can be said to “provide a historical
context.” Choice (D) is therefore the best answer. 8. The comparison in lines 53–54 (“Beauty . . . rag”)
◾ Choice (A) is incorrect because the first paragraph primarily serves to
cannot be said to “formulate a generalization.” Rather,
(A) inject a mocking tone
it gives detailed information about a specific historical
(B) introduce a controversial theory
occurrence. (C) reinforce a previous assertion
◾ Choice (B) is incorrect because no “problem” is (D) correct a factual inaccuracy
defined in the first paragraph. (E) acknowledge a noteworthy discovery
◾ Choice (C) is incorrect. The first paragraph does not
directly discuss Kepler’s fame, so it cannot be said to In lines 53–54, the author states that “Beauty soaks
“underscore a scientist’s notoriety.” reality as water fills a rag”; that is, beauty thoroughly
◾ Choice (E) is incorrect because no “hypothesis” is saturates the natural world. In the previous sentences,
questioned in the first paragraph. the author makes much the same point: “Beauty in
nature is not something that shows up only at a certain
level of complexity. Beauty is built into . . . every atomic
brick!” So the comparison in lines 53–54 functions to
“reinforce a previous assertion,” making choice (C) the
best response.

PSAT/NMSQT Official Student Guide: Prepare for the Test 13


◾ Choice (A) is incorrect because the author is not Passage 2
mocking anything in making this comparison.
◾ Choice (B) is incorrect because the author never My earliest memory of Stonehenge is, like so many
childhood memories, as much fiction as fact. I see a
suggests that it is “controversial” to claim that beauty
youngster standing at a country roadside. Larks sing and
is to be found at every level of nature. fly about. There across the plain the great stones rise and
◾ Choice (D) is incorrect because it is never stated as a (30) I run towards them, ahead of my parents—not at all, I’m
“fact” that beauty is not found at all levels of nature. afraid, as a budding scholar or an embryo poet. But at least
The author is therefore not correcting “a factual I recognize a good natural exploring place when I see one.
inaccuracy.” Climbing, scrambling, squeezing through stone pillars.
◾ Choice (E) is incorrect because the comparison in One part of my memory must be very wrong, because
lines 53–54 does not “acknowledge” a particular (35) people have not been allowed to walk up to the monument
discovery. Rather, it reiterates a general and widely as they like since well before my birth; and even in the
1930s, I am pretty sure that, though one was then free
accepted assertion about the natural world.
to wander in the central circle of stones, eight-year-old
mountaineers were not encouraged. Of one thing I am
Questions 9–12 are based on the following passages. (40) certain: my own first meeting was happy. Above all,
Stonehenge’s marvelous openness was what pleased me.
Prehistoric burial mounds and other megalithic (large stone)
My latest remembrance, on a recent clear but arctic
monuments dot the countryside in Europe and in the British Isles,
November day, is sadly different. Stonehenge stands in
many in isolated and unprotected places. The first passage below is
the fork of two busy roads, and the dominant sound in its
adapted from a history of the popular fascination with megalithic (45) present landscape is not the lark song of my memory, but
sites. The second passage is from a book about the most famous of
the rather less poetic territorial whine of the long-distance
these monuments, Stonehenge, which stands in southwest England.
truck. Visitors get to it now from a parking lot, past a
Passage 1 sunken “sales complex,” then down a tunnel under the
There is little evidence that most nineteenth-century nearest road: all this designed not to spoil the view, but the
amateur archaeologists ever did any work. In most
(50) effect is unhappily reminiscent of an underground
illustrations and paintings of the era, they are depicted in bunker. When visitors finally rise inside the wired-off
Line attitudes of elegant supervision, while sturdy workers, eager enclosure, they are promptly faced with another barrier.
(5) for the sight of buried treasure, shoveled away ancient burial The public is now forbidden to enter the central area.
mounds and stone monuments. Some antiquarians Conservation is a fine thing; yet one feels in some way
were motivated by desire for loot, others by the spirit
(55) cheated of a birthright. Everyone I had spoken to before
of scientific inquiry, or by the two combined; but coming had warned me that the new preserved-for-
whatever the motive, the results were the same. Even if a posterity Stonehenge makes a depressing visit.
(10) megalithic site were not entirely destroyed or removed in I went up to an attendant in a little wind-shelter and
the course of archaeological investigation, once it had been explained I was writing a book about Stonehenge and
disturbed and was exposed to the elements it gradually
(60) would like to walk inside the barrier.
disintegrated. As a result, many important monuments have “Are you an archaeologist?”
totally disappeared since their excavation. In addition, “No, just a writer.”
(15) the loss of prehistoric artifacts during or following their “Department of the Environment, London. By letter.”
disinterment occurred on a large scale. Some remnants Then he added, “And I can tell you you’ll be wasting your
were preserved in museums and still exist, but those that
(65) time.”
passed into private collections were commonly dispersed He looked bleakly over my shoulder at the mute clump
with no record of their histories. of stones, as a prison warden might who has successfully
(20) The technique of totally stripping sites under foiled yet another clumsy escape attempt. I didn’t really
investigation was used by one amateur archaeologist, blame him, for it was bitterly cold; and after all, who cares
Colonel Hawley, in his excavations of Stonehenge in the
(70) for mere curiosity and affection any more?
1920s. The site was dug up like a potato field. Much of it
was stripped to the bedrock, and its potential for yielding
(25) information to future investigators was destroyed forever.

14 PSAT/NMSQT Official Student Guide: Prepare for the Test


9. The author of Passage 1 refers to Colonel Hawley 10. Passage 2 as a whole can best be described as
(lines 20–25) to
(A) an account of contemporary efforts to protect
(A) draw a parallel between an archaeological expedition Stonehenge
and a military campaign (B) a contrast between idealistic expectations and
(B) reveal the extent of tourist interest in megalithic sites nostalgic reminiscences
(C) provide an example of an excavation that damaged a (C) a comparison of youthful impressions of
megalithic site Stonehenge and present-day conditions
(D) show that some amateur archaeologists were (D) a celebration of a favorite family memory
motivated by a desire for knowledge (E) a discussion of reforms needed to preserve
(E) suggest a relationship between scientific inquiry and monuments like Stonehenge
pecuniary gain
The phrase “as a whole” indicates that you should look
Carefully read the passage and the lines in question. The for the answer choice that provides the best overall
first paragraph focuses on the damage that amateur description of the passage. The first two paragraphs in
archaeologists did to megalithic sites. Lines 20–25 then Passage 2 describe the author’s “earliest memory” of
refer to archaeologist Colonel Hawley, who is said to Stonehenge, which can be characterized as “youthful
have dug up a site like a “potato field.” As a result, its impressions.” The remainder of the passage describes the
“potential for yielding information” was “destroyed author’s “latest remembrance” from a recent visit. This
forever.” What point is the author making with this portion of the text can be said to describe “present-day
reference to Colonel Hawley? Choice (C) is the best conditions.” Choice (C) describes Passage 2 “as a whole”
answer because it correctly describes the purpose of the and is therefore the best answer.
reference, which is to provide a specific illustration of ◾ Choice (A) may seem appealing since this passage
“an excavation that damaged a megalithic site.” does describe “contemporary efforts to protect
◾ Choice (A) is incorrect. Although these lines involve a Stonehenge.” But this choice does not take into
colonel, the author is not drawing a parallel with a account the first part of the passage in which the
“military campaign.” author recounts childhood experiences at Stonehenge,
◾ Choice (B) is incorrect because Colonel Hawley can’t so this is not the best response.
be considered a “tourist” in the ordinary sense of the ◾ Choice (B) is incorrect. While the first part of the
word. Moreover, Passage 1 does not discuss “tourist passage does include some “nostalgic reminiscences,”
interest in megalithic sites.” the author did not have “idealistic expectations” about
◾ Choice (D) may be appealing because some amateur returning to Stonehenge. Indeed, the author was
archaeologists probably did have a “desire for “warned” that the new Stonehenge made for “a
knowledge.” But the author doesn’t discuss Hawley’s depressing visit.”
motivations, so choice (D) is not the best answer. ◾ Choice (D) is incorrect. The first part of the passage
◾ Choice (E) is incorrect because these lines do not may describe a “favorite family memory,” but this
reveal whether or not Hawley realized any answer choice doesn’t take into account the author’s
“pecuniary,” or monetary, gain. purpose in the second part of the passage.
◾ Choice (E) is incorrect because the author doesn’t
discuss “reforms needed to preserve monuments like
Stonehenge.”
Tip: If two answer choices seem appealing, review both of
them carefully and pick the better of the two.

PSAT/NMSQT Official Student Guide: Prepare for the Test 15


11. The author of Passage 1 would most likely view the 12. Which statement best describes an important difference
conditions described in lines 51–52 in Passage 2 as a between the two passages?
sign of
(A) The first discusses the changing role of amateurs in
(A) unfortunate governmental initiatives that threaten the archaeological profession; the second describes
tourism in England a controversy between professional and non-
(B) an increased awareness of the fragility of megalithic professional archaeologists.
sites (B) The first describes the destruction of megalithic
(C) the inadequate efforts of amateur archaeologists to sites; the second shows what can be learned from a
protect Stonehenge damaged site.
(D) a willingness to allow those who are not archaeol- (C) The first describes how contemporary archaeologists
ogists to visit excavation sites have worked to preserve megalithic sites; the
(E) overdue scientific discoveries about the origin of second describes a personal encounter with a
Stonehenge single monument.
(D) The first supports the preservation of megalithic
In Passage 2, lines 51–52 describe how a series of sites; the second questions the value of complete
barriers prevent visitors from entering Stonehenge, a protection.
megalithic site. This question asks how the author of (E) The first offers a scientific perspective on megalithic
Passage 1 would view such conditions. Since Passage 1 monuments; the second discusses their financial
reveals that many megalithic sites were damaged or value as artistic artifacts.
“destroyed forever” by amateur archaeologists, it is likely
that this author would see the modern effort to protect This question requires that you understand the primary
Stonehenge as proof that people have become more arguments of both passages and the essential difference
aware of the “fragility of megalithic sites.” Choice (B), between them. The author of Passage 1 focuses on the
therefore, is the correct answer. damage done to megalithic sites and implicitly supports
◾ Choice (A) is incorrect. It is unlikely that the author the idea that such sites should be preserved. The author
of Passage 1 would regard these conditions as of Passage 2 describes a childhood experience at
“unfortunate governmental initiatives,” since they Stonehenge and compares it with a more recent visit;
serve to protect the site. lines 54–57 specifically suggest that recent conservation
◾ Choice (C) is incorrect because nothing suggests that
efforts, while a good thing, may have been taken too far.
the author of Passage 1 would regard these efforts as Choice (D), which accurately characterizes these
“inadequate” or as the work of “amateur different attitudes toward preservation, is therefore the
archaeologists.” best response.
◾ Choice (B) may be appealing because Passage 1 does
◾ Choice (D) is incorrect because this author does not
discuss whether people “who are not archaeologists” describe the destruction of megalithic sites, but it is
should be allowed to visit sites. incorrect because Passage 2 doesn’t show “what can be
◾ Choice (E) is incorrect because the conditions in lines
learned from a damaged site.”
◾ Choice (C) may be appealing because Passage 2 does
51–52 do not involve “overdue scientific discoveries
about the origin of Stonehenge.” describe a “personal encounter with a single
monument,” but it is incorrect because Passage 1
doesn’t talk about “contemporary archaeologists.”
◾ Choices (A) and (E) are incorrect because they
inaccurately characterize both of the passages.
Tip: For questions that focus on a similarity or difference
between two related passages, make sure that your answer
choice is true for both passages.

16 PSAT/NMSQT Official Student Guide: Prepare for the Test


◾ Rational and radical equations
Mathematics Section ◾ Equations of lines
◾ Absolute value
Mathematics Content Overview ◾ Direct and inverse variation
The following math concepts are covered on the ◾ Concepts of algebraic functions
PSAT/NMSQT. ◾ Newly defined symbols based on commonly used
Number and Operations (20–25%) operations
◾ Arithmetic word problems (including percent, ratio Geometry and Measurement (25–30%)
and proportion) ◾ Area and perimeter of a polygon
◾ Properties of integers (even, odd, prime numbers, ◾ Area and circumference of a circle
divisibility, etc.) ◾ Volume of a box, cube and cylinder
◾ Rational numbers ◾ Pythagorean Theorem and special properties of
◾ Sets (union, intersection, elements) isosceles, equilateral and right triangles
◾ Counting techniques ◾ Properties of parallel and perpendicular lines
◾ Sequences and series ◾ Coordinate geometry
◾ Elementary number theory ◾ Geometric visualization
Algebra and Functions (35–40%) ◾ Slope
◾ Substitution and simplifying algebraic expressions ◾ Similarity
◾ Properties of exponents ◾ Transformations
◾ Algebraic word problems Data Analysis, Statistics and Probability (10–15%)
◾ Solutions of linear equations and inequalities ◾ Data interpretation (tables and graphs)
◾ Systems of equations and inequalities ◾ Statistics (mean, median and mode)
◾ Quadratic equations ◾ Probability

Calculator Use

h You are strongly encouraged to bring a calculator h You will not be permitted to use a pocket
with you when you take the PSAT/NMSQT, even organizer; laptop or handheld electronic device;
if you don’t think you will use it. Schools are not cell phone calculator; or calculator with QWERTY
required to provide calculators. (typewriter-like) keypad, with paper tape, that makes
noise or “talks,” or that requires an electrical outlet.
h A scientific or graphing calculator is
recommended. Although not recommended, any h If you use a calculator with a large (characters
four-function calculator is permitted. 1 inch or more high) or a raised display that might
be visible to other test-takers, you will be seated at
h Bring a calculator you are comfortable using. the discretion of the test supervisor.
Don’t buy a sophisticated or new calculator just to
take the PSAT/NMSQT. h You will not be allowed to share a calculator with
other students.
h Don’t try to use your calculator on every question.
No question will require the use of a calculator. h Make sure your calculator is in good working
However, studies indicate that students who use order and that batteries are fresh. If your calculator
calculators do slightly better than students who fails during testing and you have no backup, you’ll
do not. have to complete the test without it.
h Decide how to solve each problem; then decide
whether to use a calculator.
h Take the practice test in this booklet with a
calculator at hand. This will give you an idea of
how much you are likely to use a calculator on
the day of the test.

PSAT/NMSQT Official Student Guide: Prepare for the Test 17


Mathematics Concept Review Exponents
You should be familiar with the following rules for
Become familiar with the following math concepts
exponents.
before test day. For additional test-taking tips, visit
www.collegeboard.com/psat. For all values of a, b, x, y :
Number and Operations x a ⋅ x b = x a +b ( x a )b = x a ⴢ b ( xy ) a = x a ⋅ y a

◾ Integers: . . . , −4, −3, −2, −1, 0, 1, 2, 3, 4, . . . For all values of a, b, x > 0, y > 0:
(Note: zero is neither positive nor negative.) xa ⎛ x ⎞ a xa x −a =
1
= xa−b = a
◾ Consecutive Integers: Integers that follow in xb ⎝ y⎠ y xa
a 2
b 3
sequence; for example, 22, 23, 24, 25. Consecutive Also, x b = x a . For example: x 3 = x 2 .
integers can be more generally represented by
n, n + 1, n + 2, n + 3, . . . Note: For any nonzero number x, it is true that x 0 = 1.
◾ Odd Integers: . . . , −7, −5, −3, −1, 1, 3, 5, 7, . . . . Variation
2 k + 1, . . . where k is an integer Direct Variation: The variable y is directly proportional
◾ Even Integers: . . . , −6, −4, −2, 0, 2, 4, 6, . . . , 2k, . . . to the variable x if there exists a nonzero constant k such
where k is an integer (Note: zero is an even integer.) that y = kx.
◾ Prime Numbers: 2, 3, 5, 7, 11, 13, 17, 19, . . .
Inverse Variation: The variable y is inversely
(Note: 1 is not a prime and 2 is the only even prime.)
proportional to the variable x if there exists a nonzero
◾ Digits: 0, 1, 2, 3, 4, 5, 6, 7, 8, 9
(Note: the units digit and the ones digit refer to the constant k such that y = k or xy = k .
x
same digit in a number. For example, in the number Absolute Value
125, the 5 is called the units digit or the ones digit.) The absolute value of x is defined as the distance from
x to zero on the number line. The absolute value of x is
Percent
written as x . For all real numbers x:
Percent means hundredths or number out of 100. For
40 2 ⎧ x, if x ≥ 0
example, 40 percent means 100 or 0.40 or 5 . x =⎨

⎩ x, if x < 0

Problem 1: If the sales tax on a $30.00 item is $1.80, what ⎧ 2 = 2 since 2 > 0
is the sales tax rate? For example: ⎨−2 = 2 since − 2 < 0
n
$1.80 = × $30.00
100 ⎩ 0 =0
Solution:
n = 6, so 6% is the sales tax
x rate.
Geometry and Measurement
Algebra and Functions Figures that accompany problems are intended to
Factoring provide information useful in solving the problems.
You may need to apply these types of factoring: They are drawn as accurately as possible EXCEPT when
x 2 + 2 x = x( x + 2 ) it is stated in a particular problem that the figure is
x 2 − 1 = (x + 1)( x − 1 ) not drawn to scale. In general, even when figures are
x 2 + 2 x + 1 = ( x + 1)(x + 1 ) = ( x + 1) 2
not drawn to scale, the relative positions of points and
angles may be assumed to be in the order shown. Also,
2 x 2 + 5 x − 3 = (2 x − 1)( x + 3 )
line segments that extend through points and appear
Functions to lie on the same line may be assumed to be on the
A function is a relation in which each element of same line. A point that appears to lie on a line or curve
the domain is paired with exactly one element of the may be assumed to lie on the line or curve.
range. On the PSAT/NMSQT and the SAT, unless The text “Note: Figure not drawn to scale. ” is included
otherwise specified, the domain of any function f is with the figure when degree measures may not be
assumed to be the set of all real numbers x for which accurately shown and specific lengths may not be drawn
f ( x ) is a real number. For example, if f ( x ) = x + 2, proportionally. The following examples illustrate what
the domain of f is all real numbers greater than or information can and cannot be assumed from figures.
equal to −2 . For this function, 14 is paired with 4, since
f (14 ) = 14 + 2 = 16 = 4.
Note: the symbol represents the positive, or
principal, square root. For example 16 = 4, not ± 4.
18 PSAT/NMSQT Official Student Guide: Prepare for the Test
Example 1: Properties of Parallel Lines
k

a° b° l
c° d°

w° x° m
y° z°

Since AD and BE are line segments, angles ACB and


DCE are vertical angles. Therefore, you can conclude 1. If two parallel lines are cut by a third line, the alternate
that x = y. Even though the figure is drawn to scale, interior angles are congruent. In the figure above,
you should NOT make any other assumptions without c = x and w = d
additional information. For example, you should NOT
2. If two parallel lines are cut by a third line, the
assume that AC = CD or that the angle at vertex E is a
corresponding angles are congruent. In the figure,
right angle even though they might look that way in the
a = w, c = y, b = x, and d = z
figure.
3. If two parallel lines are cut by a third line, the sum of
Example 2: the measures of the interior angles on the same side
of this line is 180°. In the figure,
c + w = 180, and d + x = 180

Angle Relationships

A question may refer to a triangle such as ABC above. 60° 50°


Although the note indicates that the figure is not drawn z° y°
to scale, you may assume the following from the figure:
◾ ABD and DBC are triangles.
◾ D is between A and C. 1. The sum of the measures of the interior angles of a
◾ A, D, and C are points on a line. triangle is 180°. In the figure above,
◾ The length of AD is less than the length of AC . x = 70 because 60 + 50 + x = 180
◾ The measure of angle ABD is less than the measure of
2. When two lines intersect, vertical angles are
angle ABC. congruent. In the figure,
You may not assume the following from the figure: y = 50

◾ The length of AD is less than the length of DC . 3. A straight angle measures 180°. In the figure,
z = 130 because z + 50 = 180
◾ The measures of angles BAD and BDA are equal.
◾ The measure of angle ABD is greater than the measure 4. The sum of the measures of the interior angles of a
of angle DBC. polygon can be found by drawing all diagonals of
◾ Angle ABC is a right angle. the polygon from one vertex and multiplying the
number of triangles formed by 180°.
Since the polygon is
divided into 3 triangles,
the sum of the measures
of its angles is 3 × 180°
or 540°.

Unless otherwise noted, the term “polygon” will be used


to mean a convex polygon, that is, a polygon in which
each interior angle has a measure of less than 180°.

PSAT/NMSQT Official Student Guide: Prepare for the Test 19


A polygon is “regular” if all sides are congruent and 5. Two polygons are similar if and only if the lengths
all angles are congruent. of their corresponding sides are in the same ratio
and the measures of their corresponding angles
Side Relationships are equal.
1. Pythagorean Theorem: In any right triangle,
a 2 + b 2 = c 2 , where c is the length of the longest
side and a and b are the lengths of the two shorter
sides.
To find the value of x, use the
Pythagorean Theorem.

x 2 = 32 + 4 2 If polygons ABCDEF and GHIJKL are similar, then AF


and GL are corresponding sides, so that
x 2 = 9 + 16
x 2 = 25 AF 10
= =
2
=
BC 18
= . Therefore x = 9 = HI
GL 5 1 HI x
x = 25 = 5
Note: AF means the line segment with endpoints A and
2. In any equilateral triangle, all sides are congruent F, and AF means the length of AF .
and all angles are congruent.
Area and Perimeter
Because the measure of the
unmarked angle is 60°, the Rectangles
measure of all angles of Area of a rectangle = length × width = l × w
the triangle are equal, and, Perimeter of a rectangle = 2(l + w ) = 2 l + 2 w
therefore, the lengths of
all sides of the triangle are Circles
equal. Area of a circle = πr 2 (where r is the radius)
x = y = 10 Circumference of a circle = 2πr = πd (where d is the diameter)

3. In an isosceles triangle, the angles opposite Triangles


congruent sides are congruent. Also, the sides 1
Area of a triangle =
2
( base × altitude )
opposite congruent angles are congruent. In the
Perimeter of a triangle = the sum of the lengths of the three sides
figures below, a = b and x = y.
Triangle Inequality: The sum of the lengths of any two sides of a
triangle must be greater than the length of the third side.

Volume
Volume of a rectangular solid (or cube) =  × w × h
( is the length, w is the width, and h iss the height)
Volume of a right circular cylinder = π r 2 h
4. In any triangle, the longest side is opposite the
(r is the radius of the base and h is the height)
largest angle, and the shortest side is opposite the
smallest angle. In the figure below, a < b < c.
Be familiar with the formulas that are provided in the
Reference Information included with the test directions.
Refer to the test directions on page 22 and in the
practice test inserted in this publication.

20 PSAT/NMSQT Official Student Guide: Prepare for the Test


Coordinate Geometry The equation of a line can be expressed as
y = mx + b, where m is the slope and b is the
3
y-intercept. Since the slope of line l is − , the
4
equation of line l can be expressed as
3
y = − x + b. Since the point (−2, 1) is on the line,
4
x = − 2 and y = 1 must satisfy the equation. Hence,
3 1
1 = + b, so b = − and the equation of line l is
2 2
1. In questions that involve the x- and y-axes, 3 1
y=− x− .
x-values to the right of the y-axis are positive 4 2
and x-values to the left of the y-axis are negative.
Data Analysis, Statistics and Probability
Similarly, y-values above the x-axis are positive
and y-values below the x-axis are negative. In
Average
an ordered pair ( x , y), the x-coordinate is written An average is a statistic that is used to summarize data.
first. Point P in the figure above appears to lie at The most common type of average is the arithmetic
the intersection of gridlines. From the figure, you mean. The average (arithmetic mean) of a list of n
can conclude that the x-coordinate of P is −2 and numbers is equal to the sum of the numbers divided by
the y-coordinate of P is 3. Therefore, the coor- n. For example the mean of 2, 3, 5, 7, and 13 is equal to
dinates of point P are (−2, 3). Similarly, you can 2 + 3 + 5 + 7 + 13
=6
5
conclude that the line shown in the figure passes
through the point with coordinates (−2, −1) and When the average of a list of n numbers is given, the sum
of the numbers can be found. For example, if the average
the point (2, 2).
of six numbers is 12, the sum of these six numbers is
rise change in y coordinates 12 × 6, or 72.
2. Slope of a line = =
run change in x coordinates The median of a list of numbers is the number in the
middle when the numbers are ordered from greatest to
least or from least to greatest. For example, the median
of 3, 8, 2, 6, and 9 is 6 because when the numbers are
ordered, 2, 3, 6, 8, 9, the number in the middle is 6.
When there are an even number of values, the median
is the same as the mean of the two middle numbers. For
example, the median of 6, 8, 9, 13, 14, and 16 is the mean
of 9 and 13, which is 11.
l The mode of a list of numbers is the number that
4 occurs most often in the list. For example, 7 is the mode
Slope of PQ = =2 of 2, 7, 5, 8, 7, and 12. The list 2, 4, 2, 8, 2, 4, 7, 4, 9, and 11
2
1 − (− 2) 3 has two modes, 2 and 4.
Slope of l = =−
−2 −2 4 Note: On the PSAT/NMSQT and the SAT, the use of
the word average refers to the arithmetic mean and is
A line that slopes upward as you go from left to right
indicated by “average (arithmetic mean).” An exception
has a positive slope. A line that slopes downward is when a question involves average rate. Questions
as you go from left to right has a negative slope. A involving median and mode will have those terms stated
horizontal line has a slope of zero. The slope of a as part of the question’s text.
vertical line is undefined.
Parallel lines have the same slope. The product of
the slopes of two perpendicular lines is −1, provided
the slope of each of the lines is defined. For example,
any line perpendicular to line l has a slope of 43 .

PSAT/NMSQT Official Student Guide: Prepare for the Test 21


Probability Mathematics Questions
Probability refers to the chance that a specific outcome
Learn about the kinds of math questions that are on
can occur. When outcomes are equally likely, probability
the test:
can be found by using the following definition:
Multiple-Choice (28 questions)
number of ways that a specific outcome can occur Student-Produced Response (10 questions)—
total number of possible outcomes
you produce and grid your own answers
For example, if a jar contains 13 red marbles and 7 green The math questions call upon the skills you have
marbles, the probability that a marble to be selected
learned in arithmetic, algebra and geometry and test
from the jar at random will be green is
how well you can use these skills. Some of the questions
7 7 are like those in your textbooks. Others require you to
= or 0.35
7 + 13 20
use your math skills in original ways to solve problems.
If a particular outcome can never occur, its probability is The ability to reason logically in a variety of situations is
0. If an outcome is certain to occur, its probability is 1. In tested throughout.
general, if p is the probability that a specific outcome will The PSAT/NMSQT does not require you to know
occur, values of p fall in the range 0 ≤ p ≤ 1. proofs of geometric theorems or more advanced
Probability may be expressed as either a decimal, a concepts from intermediate algebra.
fraction, or a ratio.

Multiple-Choice Questions
Below are the directions that you will see on the test.

Directions: For this section, solve each problem and decide which is the best of the choices given. Fill in the
corresponding oval on the answer sheet. You may use any available space in the test book for scratch work.

Tip: The test directions show you formulas for the area of a rectangle, the circumference of a circle, and other important
reference information. Before you take the test, become familiar with which formulas will be given in the directions. This
reference information will also appear in the Student-Produced Response Questions section of the test.

22 PSAT/NMSQT Official Student Guide: Prepare for the Test


3. In a competition, Scott received scores of 6, 7, 8, 9, 9, 9,
y° x° y° and 10 from the seven judges, respectively. The score of 9
is described by which of the following measures?
x° x°
y° I. The mode
II. The median
III. The average (arithmetic mean)
Note: Figure not drawn to scale.
(A) I only
(B) II only
1. In the figure above, if y = 40 what is the value of x?
(C) I and II only
(A) 60 (B) 70 (C) 80 (D) 85 (E) 90 (D) I and III only
(E) I, II, and III

Tip: Figures are always drawn to scale unless the question


says otherwise. Tip: Use educated guessing to improve your chances of
answering the question correctly if you do not know how
to solve the problem completely. Eliminate answers you
It is helpful to mark the figure with information given in know to be wrong and select from those remaining, even
the problem as shown below. if you’re not completely certain about which answer is
correct.

To answer this type of question, consider each of the


40° x° 40° statements I, II and III separately. The mode is the
x° x° most frequently used number in a list, which in this
40° case is the number 9. Therefore, I is true. At this point,
choice (B) can be eliminated from consideration and you
could guess among the choices (A), (C), (D), or (E).
You have improved your chances of answering the
Now you can visualize the solution as 3x + 120 = 360, since question correctly since you now have a 1 in 4 chance
the sum of the measures of the angles about a point is of answering the question correctly instead of a 1 in 5
360°. Solving this equation yields 3x = 240 or
chance.
x = 80. The correct answer is (C).
But to improve your chances of using educated
guessing to answer the question correctly, consider II.
2. For all values of r, let ★ r be defined as ★ r = r + 2 .
2 The median is the middle term of a list of numbers
If ★4 = x, then ★x =
when the numbers are listed in either increasing or
3 5 decreasing order. In this list of seven numbers, the
(A) 2 (B) 2 (C) (D) 3 (E) 4
2 middle number will be the fourth number, which is 9,
since the numbers are already listed in increasing order.
The symbol ★r is defined in this problem. This is not a Since II is true, choices (A) and (D) can be eliminated
definition you should have studied in one of your math from consideration. At this point, if you do not know
classes in school. Since ★ r = r +2 2 , then how to find the average of a list of seven numbers, you
could guess between choices (C) and (E), which would
4 +2 3+2 5
★4= = 3 = x. Therefore ★x = ★ 3 = = . give you a 1 in 2 chance of answering the question
2 2 2
correctly.
The correct answer is (C). The average of seven numbers can be found by
adding the numbers together and dividing by 7. In this
58 2
case, the average of the seven numbers is 7 = 8 7 .
Therefore, III is not correct, so the answer is (C).
You can also determine that the average of the
numbers does not equal 9 since the three numbers larger
than 9 are closer to 9 than the three numbers smaller
than 9. This tells you that the average will be less than 9.
For this problem, it was not necessary to determine the

PSAT/NMSQT Official Student Guide: Prepare for the Test 23


exact average; it was sufficient just to determine that the Then consider all possible three-digit numbers whose
average does not equal 9. hundreds digit is three, and then consider all possible
three-digit numbers whose hundreds digit is 4.
Questions 4–5 refer to the following table, which shows the
241 312 412
results of a survey of 50 people.
243 314 413
FAVORITE MUSIC CATEGORIES 321 421
Blues Classical Jazz 324 423
Women 11 13 6 341 431
Men 2 10 8 342 432
This systematic approach allows you to see that you have
4. What percentage of the people surveyed were women? not forgotten any numbers in your count. There are 14
numbers that satisfy the conditions in the problem. The
(A) 30% (B) 33% (C) 57% (D) 60% (E) 67% correct answer is (C).

7. The coordinates of the center of a circle are (5, 3). If AB


5. If a person is to be chosen at random from among the is a diameter of the circle and the coordinates of B are
group that favored blues, what is the probability that the (8, 5) what are the coordinates of A ?
person chosen will be a man?
1 2 2 2 1 (A) (11, 7 ) (B) (3, 1) (C) (3, 0 ) (D) (2, 1) (E) (1, 2 )
(A) 10 (B) 13 (C) 11 (D) 5 (E) 2

For question 4, the percentage of the people surveyed Tip: For a geometry problem, if a figure is not drawn, you
who were women can be found by finding the total may find it helpful to draw the figure so you can visualize
number of women surveyed (30) and dividing this the problem.
number by the total number of people surveyed (50).
30 Drawing a figure may be helpful to visualize the solution
Therefore, 50 = .60, or 60%. The correct answer is (D).
to this problem. The figure below shows the center of
For question 5, the total number of people who the circle (5, 3) and point B with coordinates (8, 5 ) .
favored blues is 11 + 2 = 13. Since 2 of these 13 people are y
2
men, there is a 13 chance that a person chosen from
B(8, 5)
this group will be a man. The correct answer is (B).
For both percentage and probability problems it is
(5, 3)
important to make sure that you are considering the
correct base. For the first problem in the set, the base is
the 50 people surveyed. For the second problem in the x
O
set, the base is the 13 people who favored blues.

6. How many different three-digit numbers greater than


Since AB is a diameter of the circle and (5, 3) will be
240 can be formed by using three different digits from the midpoint of AB , it follows that point A will have x
the set {1, 2, 3, 4 }? and y coordinates both less than the x and y coordinates
of the center of the circle.
(A) 24 (B) 16 (C) 14 (D) 12 (E) 10 Let (a, b ) be the coordinates of point A. Using the
midpoint formula, it follows that 8 + a = 5 or a = 2 and
2
To determine all possible three-digit numbers that satisfy b+5
= 3 or b = 1. The coordinates of point A are
the conditions in the problem, establish a systematic 2
approach to counting these numbers so that no numbers therefore (2,1) and the correct answer is (D).
are forgotten in your count. You can also solve this problem by considering that B
Since the three-digit numbers must be greater than is 3 units over and 2 units up from the center. Therefore,
240, the hundreds digit cannot be 1. If the hundreds point A will be 3 units to the left and 2 units down from
digit is 2, then the tens digit must equal 4 in order for (5, 3) or the coordinates of point A will be (5 − 3, 3 − 2 ) or
the number to be greater than 240. That leaves 1 or 3 for (2, 1) .
the units digit, so 241 and 243 are both greater than 240.

24 PSAT/NMSQT Official Student Guide: Prepare for the Test


x f (x) C ( d ) = kd + t
0 4
1 2
10. The Nelson family has discovered that the cost C of their
family vacation is a constant k times the length d, in days,
2 0
of the vacation added to the cost t of transportation to get
3 1
to and from their vacation spot, as shown by the function
4 3
above. The Nelson family had to shorten their planned
vacation by 3 days. How much less did the shortened
8. According to the table above, for what value of x does vacation cost than the original planned vacation?
f (x ) = x + 1 ?
(A) 3d (B) 3k (C) 3t (D) d ( k − 3) (E) k ( d − 3)
(A) 0 (B) 1 (C) 2 (D) 3 (E) 4

This question requires you to understand f ( x ) notation Tip: If you have difficulty determining what is being
and to be able to use a table that gives five x values and asked in a question, look at the answer choices before you
the five corresponding f ( x ) values. For example, the first begin working the problem. The answer choices often will
row in the table tells you that f (0 ) = 4. help you focus on what you are supposed to look for in
the problem.
To solve this problem, you need to locate the row
in the table in which the number in the right-hand Suppose that the original planned vacation was for
column, f ( x ) , is one more than the number in the left- d1 days. Then the cost of this vacation, based on the
hand column, x. The second row, with 1 in the left-hand formula given in the problem, would be C (d1 ) or kd1 + t.
column and 2 in the right-hand column, is the only one The cost of the shortened vacation would then be
that satisfies this condition. When x = 1, f ( x ) = 2, which C ( d1 − 3) or k ( d1 − 3) + t since this vacation will be 3 days
is x + 1 in this case. The correct answer is (B). shorter. The difference between C (d1 ) and C (d1 − 3) can
be written as (kd1 + t ) − [k (d1 − 3) + t ], or 3k. The correct
answer is (B).
9. If x and y are integers such that 1 < x < 6 and 2 < y < 5,
what is the least possible value of x + y ? Another way to solve this problem is for a student
to realize that the transportation cost t to the vacation
(A) −11 (B) −10 (C) −9 (D) 3 (E) 5 spot will be the same no matter how long the vacation.
Therefore, the transportation cost can be ignored and
This question involves the symbol for absolute value. the shortened vacation will cost 3k (the constant k times
The absolute value of x (written as x ) is x when x is the 3-day difference) less than the original planned
positive and −x when x is negative. (Recall that the vacation.
negative of a negative number is positive.) When x is
The PSAT/NMSQT could contain 1 or 2 questions
zero, its absolute value is zero. Here are some examples
involving functions as models as illustrated by this
of absolute value: 11 = 11, and −11 = 11, and 0 = 0.
question. A model is a mathematical equation, inequality,
In this question, since x is an integer and 1 < x < 6 , or expression that describes a real-world situation.
the possible values of x are 2, 3, 4, and 5. If x = 2, then
x can be either 2 or −2. This is true because the absolute
value of 2 is 2 and the absolute value of −2 is also 2. The
possible values of x are −5, −4, −3, −2, 2, 3, 4, 5. Similarly,
the possible values of y are −4, −3, 3, 4. To make the
value of x + y as small as possible, use the smallest
possible values for x and y, which are −5 for x and −4
for y. The least possible value for x + y is −9. The correct
answer is (C).

PSAT/NMSQT Official Student Guide: Prepare for the Test 25


Student-Produced Response Questions these directions carefully so you will know them
These questions do not include any answer choices. thoroughly before the test. You do not lose any points
You are required to solve 10 problems and enter your for a wrong answer to a Student-Produced Response
answers in the grids provided on the answer sheet. Read Question.

Note: You should grid only one of the possible correct


Tip: Learn these gridding rules, since misgridding is
the most common way to lose points when you know the
answers. There are three possible positions for gridding a
correct answer. 2-digit integer—in the center of the grid, to the far right,
or to the far left.
11. Set S consists of all multiples of 3 between 10 and 25. Set
OR OR
T consists of all multiples of 4 between 10 and 25. What
⁄ ⁄ ⁄ ⁄ ⁄ ⁄
is one possible number that is in set S but not in set T ?
. . . . . . . . . . . .
Set S can be written as {12, 15, 18, 21, 24 } and set T can 0 0 0 0 0 0 0 0 0
1 1 1 1 1 1 1 1 1 1 1 1
be written as {12, 16, 20, 24 }. The numbers that are in set S 2 2 2 2 2 2 2 2 2 2 2 2
but not in set T are 15, 18, and 21. Any of these three 3 3 3 3 3 3 3 3 3 3 3 3

numbers can be gridded since the question asks for one 4 4 4 4 4 4 4 4 4 4 4 4


5 5 5 5 5 5 5 5 5 5 5 5
possible number. A two-digit number can be gridded as
6 6 6 6 6 6 6 6 6 6 6 6
shown to the right. 7 7 7 7 7 7 7 7 7 7 7 7
8 8 8 8 8 8 8 8 8 8 8 8
9 9 9 9 9 9 9 9 9 9 9 9

26 PSAT/NMSQT Official Student Guide: Prepare for the Test


Note: The grid will not accommodate mixed numbers
Tip: Remember, only what is gridded will count, so such as 4 1/2. If you grid 41/2, the result will be
always double check your grids and make sure each 41 1
column grid matches the number that you have written interpreted as 2 , not 4 2 . So if you obtain a mixed
above the column. number as an answer, you will need to change the mixed
number to an improper fraction and enter the improper
fraction on the grid or enter its decimal equivalent.
1
The mixed number 4 2 can be gridded as 9/2 or 4.5, as
A B C D E shown in the left column. It is not necessary to reduce
a fraction to lowest terms unless the fraction will not fit
on the grid.

12. b
In the figure above, B is the midpoint of AC and D is 13. If x = c and x = 3, what is the value of cy ?
the midpoint of CE. If the length of AE is 9, what is the y bx
length of BD?
x c 3 c
Since y = bx and x = 3 then y = 3b , or by cross
b substitute
multiplying, 9b = cy. To find the value of cy
Tip: Throughout the test, figures will be drawn to ,
scale unless otherwise indicated. (See directions at the 9b for cy in this expression. Therefore b = b = 1 .
beginning of each math section.) cy 9b 9
The fraction 1/9 or its decimal equivalent of .111 can
It is helpful to put as much information in a given figure be gridded as shown below. (Note that you must grid the
as possible. Since B is the midpoint of AC , then AB = BC most accurate decimal value that the grid can
and these lengths can both be labeled as x. Since D is the accommodate. Therefore, .1 or .11 will NOT be counted
midpoint of CE, then CD = DE and these lengths can as correct.)
both be labeled as y.
x x y y
⁄ ⁄ ⁄
A B C D E
. . . . . . . .
0 0 0 0 0 0

The question asks for the length of BD, which from 1 1 1 1 1 1 1 1


2 2 2 2 2 2 2 2
the figure is the value of x + y. Since AE = 9, 2x + 2y = 9
3 3 3 3
or x + y = 29 . The fraction 9/2 or its decimal equivalent,
3 3 3 3
4 4 4 4 4 4 4 4

4.5, can be gridded as shown below. 5 5 5 5 5 5 5 5


6 6 6 6 6 6 6 6
7 7 7 7 7 7 7 7

OR 8 8 8 8 8 8 8 8
9 9 9 9 9 9 9 9
⁄ ⁄ ⁄
. . . . . . . .
0 0 0 0 0 0 Tip: Know the rules for gridding repeating decimals.
1 1 1 1 1 1 1 1
2 2 2 2 2 2 2 2
3 3 3 3 3 3 3 3
4 4 4 4 4 4 4 4
5 5 5 5 5 5 5 5
6 6 6 6 6 6 6 6
7 7 7 7 7 7 7 7
8 8 8 8 8 8 8 8
9 9 9 9 9 9 9 9

Tip: Know the rules for gridding mixed numbers before


taking the test.

PSAT/NMSQT Official Student Guide: Prepare for the Test 27


Line  goes through the origin (0, 0) and through the
y
k
point (6, 2). Since slope is “rise over run” (change in y
over change in x), the slope of  is 2 − 0 , which is
6−0
1
ᐉ equivalent to 3 . Since k is perpendicular to  , the slope
(6, 2)
1
of k is the negative reciprocal of 3 , which is −3. Two
x
O points on line k are (6, 2) and (4, n). The slope of k can be
2−n
expressed as 6 − 4 or 2 −2 n . Since this slope is −3, you
14. In the xy-coordinate system above, the lines  and k are can form the equation 2 − n = − 3, which simplifies to
perpendicular. If the point (4, n) is on line k, what is the 2
2 − n = − 6, or n = 8. You can check that this is a
value of n ?
reasonable answer by approximating the location
To solve this problem it is helpful to know the (4, 8) on the figure and seeing that it does appear to lie
relationship between the slopes of two perpendicular on k. The number 8 can be gridded in four ways, as
lines. Whenever two nonvertical lines are perpendicular, shown below.
their slopes are the negative reciprocals of each other.
That is, the product of their slopes is −1.

OR OR OR
⁄ ⁄ ⁄ ⁄ ⁄ ⁄ ⁄ ⁄

. . . . . . . . . . . . . . . .
0 0 0 0 0 0 0 0 0 0 0 0

1 1 1 1 1 1 1 1 1 1 1 1 1 1 1 1

2 2 2 2 2 2 2 2 2 2 2 2 2 2 2 2

3 3 3 3 3 3 3 3 3 3 3 3 3 3 3 3

4 4 4 4 4 4 4 4 4 4 4 4 4 4 4 4

5 5 5 5 5 5 5 5 5 5 5 5 5 5 5 5

6 6 6 6 6 6 6 6 6 6 6 6 6 6 6 6

7 7 7 7 7 7 7 7 7 7 7 7 7 7 7 7

8 8 8 8 8 8 8 8 8 8 8 8 8 8 8 8

9 9 9 9 9 9 9 9 9 9 9 9 9 9 9 9

28 PSAT/NMSQT Official Student Guide: Prepare for the Test


Writing Skills Section Questions measure language skills you have acquired
throughout your life, rather than knowledge acquired in
a particular course. The PSAT/NMSQT does not require
Learn about the kinds of writing skills questions that
you to define or use grammatical terms or to write
are on the test:
an essay. It does not test spelling or capitalization. All
Improving Sentences (20 questions)
questions are multiple-choice.
Identifying Sentence Errors (14 questions)
Questions relate to characteristics of effective
Improving Paragraphs (5 questions)
writing. Effective writing (1) is consistent, (2) expresses
Writing skills questions measure your ability to identify ideas logically, (3) is precise and clear and (4) follows
appropriate expressions in standard written English, conventions. Some questions test your ability to
detect faults in usage and structure, choose effective recognize effective writing.
revisions to sentences and paragraphs, and recognize The chart below illustrates common writing problems
appropriate writing strategies. covered by questions in the writing skills section.

Characteristics of Effective Writing


Writing skills questions focus on common problems associated with four characteristics of effective writing. Examples are below.

Writing Characteristic Sentence Illustrating the Problem Should Be ...

1. Being Consistent
Sequence of tenses After he broke his arm, he is home for two weeks. After he broke his arm, he was home for two weeks.
Avoiding pronoun shift If one is tense, they should try to relax. If one is tense, one should try to relax.
Parallelism She likes to ski, plays tennis, and flying hang She likes to ski, play tennis, and fly hang gliders.
gliders.
Noun agreement Carmen and Sarah want to be a pilot. Carmen and Sarah want to be pilots.
Pronoun reference Several people wanted the job, and he or she filled Several people wanted the job, and they filled out
out the required applications. the required applications.
Subject-verb agreement There is eight people on the shore. There are eight people on the shore.

2. Expressing Ideas Logically


Coordination and Tawanda has a rash, and she is probably allergic to Tawanda has a rash; she is probably allergic to
subordination something. something.
Logical comparison Nathan grew more vegetables than his neighbor’s Nathan grew more vegetables than his neighbor
garden. grew.
Modification and word order Barking loudly, the tree had the dog’s leash Barking loudly, the dog wrapped its leash around
wrapped around it. the tree.
3. Being Clear and Precise
Avoiding ambiguous and In the newspaper they say that few people voted. The newspaper reported that few people voted.
vague pronouns
Diction He circumvented the globe on his trip. He circumnavigated the globe on his trip.
Avoiding wordiness There are many problems in the contemporary There are many problems in the contemporary
world in which we live. world.
Avoiding improper If your car is parked here while not eating in the If you park here and do not eat in the restaurant,
modification restaurant, it will be towed away. your car will be towed away.

4. Following Conventions
Pronoun case He sat between you and I at the stadium. He sat between you and me at the stadium.
Idiom Natalie had a different opinion towards her. Natalie had a different opinion about her.
Comparison of modifiers Of the sixteen executives, Naomi makes more Of the sixteen executives, Naomi makes the most
money. money.
Sentence fragment Fred having to go home early. Fred has to go home early.

Some questions contain no errors, requiring you to recognize effective writing.

PSAT/NMSQT Official Student Guide: Prepare for the Test 29


Improving Sentences composer, turning out countless jingles for short-lived
These questions require that you choose the best, most television commercials, was tormented by her sense of
effective form of a sentence. Below are the directions isolation from serious music.
you will see on the test. ◾ Choice (B) fails to remedy the error because it does
not provide the main verb that the original lacks.
◾ Choices (D) and (E) do provide main verbs—found
The following sentences test correctness and
effectiveness of expression. Part of each sentence and was tormented, respectively—but each also makes
or the entire sentence is underlined; beneath each an inappropriate use of a semicolon. In each case, the
sentence are five ways of phrasing the underlined clause that precedes the semicolon lacks a main verb
material. Choice A repeats the original phrasing; and so fails to express a complete thought.
the other four choices are different. If you think the
original phrasing produces a better sentence than 2. Chaplin will not be remembered for espousing radical
any of the alternatives, select choice A; if not, select causes any more than they will remember Wayne for
one of the other choices. endorsing conservative political candidates.
In making your selection, follow the requirements
(A) any more than they will remember Wayne
of standard written English; that is, pay attention to
(B) as will Wayne not be remembered
grammar, choice of words, sentence construction,
(C) any more than Wayne will be remembered
and punctuation. Your selection should result in the
(D) just as they will not remember Wayne
most effective sentence—clear and precise, without
(E) no more than Wayne will be remembered
awkwardness or ambiguity.
EXAMPLE:
The original sentence is incorrect because they will
Laura Ingalls Wilder published her first book remember Wayne is not structurally parallel to Chaplin
and she was sixty-five years old then. will not be remembered: the construction shifts
(A) and she was sixty-five years old then awkwardly from passive to active voice. It also makes
(B) when she was sixty-five vague use of the pronoun they, which has no logical
(C) at age sixty-five years old antecedent in the sentence. Choice (C) is the correct
(D) upon the reaching of sixty-five years answer: it remedies the problem of the unclear pronoun
(E) at the time when she was sixty-five reference and clearly establishes the parallel structure
needed to compare Chaplin with Wayne. The corrected
A C D E
sentence reads: Chaplin will not be remembered for
espousing radical causes any more than Wayne will be
remembered for endorsing conservative political
1. The young composer, turning out countless jingles for candidates.
short-lived television commercials, and tormented by her ◾ Choice (B) is confused in word order and logic.
sense of isolation from serious music. ◾ Choice (D), like choice (A), suffers from unclear
pronoun reference (they) and lack of parallel
(A) commercials, and tormented by
structure.
(B) commercials, tormented by
◾ In choice (E), no more than results in a double
(C) commercials, was tormented by
(D) commercials; she found torment in negative.
(E) commercials; she was tormented by Tip: Compare the structures of phrases that are joined by
conjunctions like and, but, or, than, and as. Parallel
The problem with the original sentence is that it is phrasing is generally preferable to nonparallel phrasing.
grammatically incomplete—a mere sentence fragment
and not a true sentence at all. It contains no main verb— 3. Being as she is a gifted storyteller, Linda Goss is an expert
only the participles turning and tormented—and so does at describing people and places.
not express a complete thought. Of the five choices, only
(C) produces a grammatically acceptable sentence. (A) Being as she is a gifted storyteller
Changing and to was before the participle tormented (B) In being a gifted storyteller
creates a main verb, was tormented, that combines with (C) A gifted storyteller
the subject The young composer to express a complete (D) Although she is a gifted storyteller
thought. The corrected sentence reads: The young (E) Telling stories giftedly

30 PSAT/NMSQT Official Student Guide: Prepare for the Test


The introductory clause of the original sentence, choice This sentence contains a logical comparison error. In
(A), which reproduces it, is wordy—weighed down by choice (D) the area of an island is compared with
unnecessary verbiage. Choice (C) is the correct answer. Switzerland itself rather than with the area of
With an economy of means, A gifted storyteller Switzerland. The corrected sentence reads: The huge
succinctly modifies the proper noun that follows (Linda Amazon River, which stretches across Brazil, has many
Goss). The corrected sentence reads: A gifted storyteller, islands; one of these is so large that its area exceeds
Linda Goss is an expert at describing people and places. that of Switzerland.
◾ Choice (B) contains the unnecessary and unidiomatic ◾ There is no error in choice (A) or in choice (B). The
In being. singular verbs stretches and has agree with the
◾ Choice (D) is grammatically acceptable, but illogical: sentence’s singular subject, Amazon River.
Although suggests a contrast of ideas where none ◾ There is no error in choice (C). The phrase one of
exists. these properly refers to a particular island.
◾ Choice (E) is unacceptably awkward; what it means to
convey should have been expressed as Gifted at telling Tip: Comparisons should always be between things of a
stories. similar nature. Beware of sentences that compare a part
of one thing with the whole of another.
Tip: Phrases involving the word being are often (though
not always) awkward or wordy and may usually be more 5. Crabs living in polluted waters will come
elegantly or concisely formulated in some other way. A
in contact with large numbers of disease-causing
B
Identifying Sentence Errors
Questions of this type ask you to find errors in sentences microorganisms because it feeds by filtering
C D
provided. Below are the directions you will see on the
test. nutrients from water. No error
E

The following sentences test your ability to recognize This sentence contains a noun-pronoun agreement error
grammar and usage errors. Each sentence contains at (C). The pronoun it is singular, while the noun to
either a single error or no error at all. No sentence
which it presumably refers, Crabs, is plural. The
corrected sentence reads: Crabs living in polluted waters
contains more than one error. The error, if there is one,
will come in contact with large numbers of disease-
is underlined and lettered. If the sentence contains
causing microorganisms because they feed by filtering
an error, select the one underlined part that must be
nutrients from water.
changed to make the sentence correct. If the sentence is
◾ There is no error in choice (A). The words living in
correct, select choice E. In choosing answers, follow the
begin a phrase that appropriately modifies the noun
requirements of standard written English.
Crabs.
EXAMPLE: ◾ There is no error in choice (B). The phrase in contact
with idiomatically complements the verb come.
The other delegates and him immediately ◾ There is no error in choice (D). The words by filtering
A B C properly introduce a phrase that modifies the verb feeds.
accepted the resolution drafted by the
D Tip: When you encounter a pronoun (like he, she, it,
him, her, they, them, his, hers, its, or theirs), ask yourself
neutral states. No error A C D E what it refers to and whether it is singular or plural.
E
What the pronoun refers to (its antecedent) must always
match the pronoun in number (singular or plural).
4. The huge Amazon River, which stretches across Brazil,
A
has many islands; one of these is so large that its area 6. Whatever price the company finally sets for the fuel
B C A
exceeds Switzerland . No error will probably be determined as much by politics
D E B C
as by a realistic appraisal of the market. No error
D E

PSAT/NMSQT Official Student Guide: Prepare for the Test 31


There is nothing wrong with this sentence. The correct (8) Through the letters a modern reader can sense
answer is choice (E), No error. their anxieties about rebellious sons and daughters,
◾ In choice (A), the adverb finally appropriately belligerent neighbors, outbreaks of plague, and shortages
qualifies the verb sets. of certain foods and textiles. (9) Unbelievably, there is a
◾ In choice (B), the adverb probably is correctly placed 1470 love letter. (10) The man who wrote it ends “I beg
between the helping verb will and the main verb be. you, let no one see this letter. (11) As soon as you have
◾ In choice (C), the phrase as much by properly initiates read it, burn it, for I would not want anyone to see it.”
a comparison that continues with the words as by (12) I was sitting on the front porch with bare feet on the
later in the sentence. hottest afternoon in July and I read that with a shiver. (13)
◾ In choice (D), the preposition of is idiomatically used I had been part of a centuries-old secret.
to complement the noun appraisal.
7. Which of the following sentences would be most logical
Tip: Choose A, B, C, or D only if you think the sentence to insert before sentence 4?
contains a grammar or usage error. DO NOT choose A,
B, C, or D, simply because you can think of an alternate (A) I first came across these letters while browsing in a
phrasing for one of the underlined portions of the library.
sentence. If you find no grammar or usage error in the (B) No, I am not dreaming; I have been reading them.
sentence, choose E. (C) On the contrary, microphones are a recent invention.
(D) Obviously, a library can open the door to mystery.
Improving Paragraphs (E) However, letters are not the oldest form of
communication.
Questions of this kind ask you to make choices about
improving the logic, coherence, or organization in a
flawed passage. Here are the directions that you will see This question asks you to improve the transition
on the test. between the first paragraph and the second. Sentence 3
refers to letters; sentence 4 refers to a book containing
The following passage is an early draft of an essay. Some letters. Choice (A), which indicates that the author of the
parts of the passage need to be rewritten. essay discovered the letters in a library (where, of course,
books are kept), provides an effective transition to
Read the passage and select the best answers for sentence 4, which refers to a book of letters the author
the questions that follow. Some questions are about found. The correct answer, then, is choice (A).
particular sentences or parts of sentences and ask ◾ Choice (B) is inappropriate because it provides a
you to improve sentence structure or word choice. response to a question that has not been raised.
Other questions ask you to consider organization ◾ Choice (C) is incorrect because the reference to
and development. In choosing answers, follow the microphones occurs in sentence 2, not in sentence 3.
requirements of standard written English. If (C) were inserted before sentence 4, it would
disruptively return to a subject that sentence 3 moves
Questions 7–10 are based on the following passage. away from.
◾ Choice (D) is a generalization that adds nothing to the
(1) This summer I felt as if I were listening in on the
essay, particularly as the writer does not indicate the
Middle Ages with a hidden microphone. (2) No, there were
source of the book.
no microphones in those days. (3) But there were letters,
◾ Choice (E), too, is inappropriate, as the observation
and sometimes these letters speak to me like voices from
very long ago.
that letters are not the oldest form of communication
(4) A book I found contained selected letters from
is irrelevant to the general theme of the essay.
five generations of a family. (5) The Pastons, who lived in a
remote part of England over 500 years ago.
Tip: When considering which sentence to insert at a
particular point in a passage, be sure to reread the
(6) Getting anywhere in the Middle Ages was
sentence that comes immediately before and the sentence
really hard, with deep rivers and few bridges and sudden
that comes immediately after the point indicated.
snowstorms coming on in the empty lands between
settlements. (7) An earl rebelled in London, so that a
messenger rode for days to tell the distant head of the
Paston family of a feared civil war.

32 PSAT/NMSQT Official Student Guide: Prepare for the Test


8. In context, which is the best version of the underlined when an earl rebelled in London. The best version reads:
portions of sentences 4 and 5 (reproduced below)? Once, when an earl rebelled in London, a messenger
rode for days to tell the distant head of the Paston family
A book I found contained selected letters from five of a feared civil war.
generations of a family. The Pastons, who lived in a
◾ Choice (A) is unacceptable because it leaves the
remote part of England over 500 years ago.
sentence without a smooth transition.
(A) (as it is now) ◾ Choice (B), like Choice (A), provides no transitional
(B) a family. The Pastons, living phrase to link sentence 7 with sentence 6.
(C) a family; it was the Pastons living ◾ Choice (C) is unsatisfactory because of the ambiguous
(D) the Paston family, who lived phrase with a rebelling earl in London.
(E) the family named Paston and living
◾ Choice (D) is wrong because it employs the present
tense (rebels) where the context clearly calls for the past.
On its own, sentence 5 is grammatically incomplete and
therefore needs to be incorporated into the preceding Tip: When revising a sentence in a passage, consider not
sentence. Choice (D) effectively joins the two sentences, only whether the revised sentence is grammatically correct
appropriately subordinating the second clause through but also how the revised sentence fits into the context.
the use of who. The best version reads: A book I found
contained selected letters from five generations of the 10. In context, which is the best revision to make to sentence
Paston family, who lived in a remote part of England 8 (reproduced below)?
over 500 years ago.
Through the letters a modern reader can sense their
◾ Choice (A) is unacceptable because it leaves sentence
anxieties about rebellious sons and daughters, belligerent
5 incomplete. neighbors, outbreaks of plague, and shortages of certain
◾ Choice (B) merely substitutes a participial phrase
foods and textiles.
(living . . .) for a relative clause (who lived . . .), leaving
sentence 5 without a main clause to express a (A) Insert “one’s reading of ” after “Through”.
complete thought. (B) Change “their” to “the Pastons’”.
(C) Change “sense” to “record”.
◾ Choice (C) joins sentences 4 and 5 with a semicolon
(D) Delete some of the examples.
but introduces an ambiguous pronoun (it) into the
(E) Insert “etc.” after “textiles”.
second clause.
◾ Choice (E) ineffectively converts sentence 5 into an
In sentence 8, it is unclear to whom the plural pronoun
awkwardly coordinated participial phrase (and living their refers. Even in the preceding sentence, only single
in . . .) rather than a subordinate clause (who lived in . . .). individuals are mentioned: the earl, the messenger, and
the head of the Paston family. Choice (B) resolves the
9. In context, which of the following is the best version of
the underlined portion of sentence 7 (reproduced below)?
uncertainty by making clear that the anxieties referred
An earl rebelled in London, so that a messenger rode to are those of the Pastons. The correct sentence reads:
for days to tell the distant head of the Paston family of Through the letters a modern reader can sense the
a feared civil war. Pastons’ anxieties about rebellious sons and daughters,
belligerent neighbors, outbreaks of plague, and shortages
(A) (As it is now)
of certain foods and textiles.
(B) An earl had rebelled in London, so
◾ Choices (A), (C), (D), and (E) all fail to address the
(C) For example, with a rebelling earl in London
(D) While an earl rebels in London, unclear pronoun reference and, for that reason alone,
(E) Once, when an earl rebelled in London, are unsatisfactory.
◾ Choice (A) adds nothing of value to the existing
Sentence 7 presents an example illustrating the general sentence.
statement in sentence 6 about the difficulty of travel ◾ Choice (C) results in an illogical sentence: modern
during the Middle Ages. However, it fails to provide a readers cannot “record” anxieties in letters written
transitional phrase that would signal this move from hundreds of years ago.
general to particular. Choice (E) appropriately
◾ Choice (D) unwisely deletes some examples, all of
introduces the transitional adverb Once to indicate that
which are distinct and evocative.
what follows is an example, and it effectively relegates
◾ Choice (E) adds no specific information to the
the less pertinent of two details to a subordinate clause:
sentence.

Now go try the PSAT/NMSQT practice test!


33 PSAT/NMSQT Official Student Guide: Prepare for the Test PSAT/NMSQT Official Student Guide: Prepare for the Test 33

You might also like